The Neonatal Client Flashcards

1
Q
Neonatal Care
1. A primiparous woman has recently given birth to a term infant. Priority teaching for the client
includes information on:
1. Sudden Infant Death Syndrome (SIDS).
2. Breast-feeding.
3. Infant bathing.
4. Infant sleep-wake cycles.
A

Neonatal Care
1. 2. Breast-fed infants should eat within the first hour of life and approximately every 2 to 3
hours. Successful breast-feeding will likely require sustained support, encouragement, and instruction
from the nurse. Information on SIDS, infant bathing, and sleep-wake cycles are also important topics
for the new parent, but can be done at any time prior to discharge.
CN: Health promotion and maintenance; CL: Analyze

How well did you know this?
1
Not at all
2
3
4
5
Perfectly
2
Q
  1. A newborn who is 20 hours old has a respiratory rate of 66, is grunting when exhaling, and has
    occasional nasal flaring. The newborn’s temperature is 98°F (36.6°C); he is breathing room air and is
    pink with acrocyanosis. The mother had membranes that were ruptured 26 hours before birth. Based
    on these data, the nurse should include which of the following in the management of the infant’s care?
  2. Continue recording vital signs, voiding, stooling, and eating patterns every 4 hours.
  3. Place a pulse oximeter and contact the primary health care provider for a prescription to draw
    blood cultures.
  4. Arrange a transfer to the neonatal intensive care unit with diagnosis of possible sepsis.
  5. Draw a complete blood count (CBC) with differential and feed the infant.
A
    1. The concern with this infant is sepsis based on prolonged rupture of membranes before birth.
      Blood cultures would provide an accurate diagnosis of sepsis, but will take 48 hours from the time
      drawn. Frequent monitoring of infant vital signs, looking for changes, and maintaining contact with the
      parents is also part of care management while awaiting culture results. Continuing with vital signs,
      voiding, stooling, and eating every 4 hours is the standard of care for a normal newborn, but a
      respiratory rate greater than 60, grunting, and occasional flaring are not normal. Although not normal,
      the need for the intensive care unit is not warranted as newborns with sepsis can be treated with
      antibiotics at the maternal bedside. The CBC does not establish the diagnosis of sepsis but the
      changes in the WBC levels can identify an infant at risk. Many experts suggest that waiting until an
      infant is 6 to 12 hours old to draw a CBC will give the most accurate results.
      CN: Reduction of risk potential; CL: Synthesize
How well did you know this?
1
Not at all
2
3
4
5
Perfectly
3
Q
  1. A neonate is born by primary cesarean section at 36 weeks’ gestation. The temperature in the
    birthing room is 70°F (21.1°C). To prevent heat loss from convection, which action should the nurse
    take?
  2. Dry the neonate quickly after birth.
  3. Keep the neonate away from air conditioning vents.
  4. Place the neonate away from outside windows.
  5. Prewarm the bed.
A
    1. The neonate should be kept away from drafts, such as from air conditioning vents, which may
      cause heat loss by convection. Evaporation is one of the most common mechanisms by which the
      neonate will lose heat, such as when the moisture on the newly born neonate’s body is converted to
      vapor. Radiation is heat loss between solid objects that are not in contact with one another such as
      walls and windows. Conduction is when heat is transferred between solid objects in contact with one
      another, such as when a neonate comes in contact with a cold mattress or scale.
      CN: Reduction of risk potential; CL: Synthesize
How well did you know this?
1
Not at all
2
3
4
5
Perfectly
4
Q
  1. The primary health care provider prescribes ampicillin 100 mg/kg/dose for a newly admitted
    neonate. The neonate weighs 1,350 g. How many milligrams should the nurse administer?
    _______________ mg.
A
4. 135 mg
The recommended dose of ampicillin for a neonate is 100 mg/kg/dose. First, determine the  neonate's weight in kilograms, and then multiply the kilograms by 100 mg. The nurse should use this formula:
1,000 g = 1 kg 
1,350 g = 1.35 kg
100 mg x 1.35 kg = 135 mg / kg

CN: Pharmacological and parenteral therapies; CL: Apply

How well did you know this?
1
Not at all
2
3
4
5
Perfectly
5
Q
  1. A neonate born at 30 weeks’ gestation and weighing 2,000 g is admitted to the neonatal
    intensive care unit. What nursing measure will decrease insensible water loss in a neonate?
  2. Bathing the baby as soon after birth as possible.
  3. Use of eye patches with phototherapy.
  4. Use of humidity in the incubator.
  5. Use of a radiant warmer.
A
    1. Adding humidity to the incubator adds moisture to the ambient air, which helps to decrease
      the insensible water loss. Bathing and the use of eye patches has no impact on insensible water loss.
      The use of a radiant warmer will increase the insensible water loss by drawing moisture out of the
      skin.
      CN: Reduction of risk potential; CL: Synthesize
How well did you know this?
1
Not at all
2
3
4
5
Perfectly
6
Q
  1. A septic preterm neonate’s IV was removed due to infiltration. While restarting the IV, the
    nurse should carefully assess the neonate for:
  2. Fever.
  3. Hyperkalemia.
  4. Hypoglycemia.
  5. Tachycardia.
A
    1. Neonates that are septic use glucose at an increased rate. During the time the IV is not
      infusing, the neonate is using the limited glucose stores available to a preterm neonate and may
      deplete them. Hypoglycemia is too little glucose in the blood; without the constant infusion of IV
      glucose, hypoglycemia will result. Fevers and hyperkalemia are not related to glucose levels.
      Tachycardia is the result of untreated hypoglycemia.
      CN: Reduction of risk potential; CL: Analyze
How well did you know this?
1
Not at all
2
3
4
5
Perfectly
7
Q
  1. The nurse makes a home visit to a 3-day-old full-term neonate who weighed 3,912 g (8 lb, 10
    oz) at birth. Today the neonate, who is being bottle-fed, weighs 3,572 g (7 lb, 14 oz). Which of thefollowing instructions should the nurse give to the mother?
  2. Continue feeding every 3 to 4 hours since the weight loss is normal.
  3. Contact the primary health care provider if the weight loss continues over the next few days.
  4. Switch to a soy-based formula because the current one seems inadequate.
  5. Change to a higher-calorie formula to prevent further weight loss.
A
    1. This 3-day-old neonate’s weight loss falls within a normal range, and therefore no action is
      needed at this time. Full-term neonates tend to lose 5% to 10% of their birth weight during the first
      few days after birth, most likely because of minimal nutritional intake. With bottle-feeding, the
      neonate’s intake varies from one feeding to another. Additionally, the neonate experiences a loss of
      extracellular fluid. Typically, neonates regain any weight loss by 7 to 10 days of life. If the weight
      loss continues after that time, the primary health care provider should be called.
      CN: Health promotion and maintenance; CL: Synthesize
How well did you know this?
1
Not at all
2
3
4
5
Perfectly
8
Q
  1. Commercial formulas contain 20 calories per 30 mL. A 1-day-old infant was fed 45 mL at 2 AM , 5:30
    AM , 8 AM , 11 AM , 2 PM , 4:30 PM , 8 PM , and 10:30 PM . What is the total amount of calories the infant
    received today?
    ______________ calories.
A
  1. 240 calories
    8 feedings x 45 ml per feeding equals 360 ml
    360 ml x 20 cal / 30 ml = 240 cal

CN: Basic care and comfort; CL: Apply

How well did you know this?
1
Not at all
2
3
4
5
Perfectly
9
Q
  1. A healthy neonate was just born in stable condition. In addition to drying the infant, what is the
    preferred method to prevent heat loss?
  2. Placing the infant under a radiant warmer.
  3. Wrapping the infant in warmer blankets.
  4. Applying a knit hat.
  5. Placing the infant skin-to-skin on the mother.
A
    1. Placing an infant on a mother’s bare chest or abdomen facilitates transition to extrauterine
      life and is the preferred method of thermoregulation for stable infants. A radiant warmer should be
      used if an infant is unstable and needs medical intervention. Blankets may be placed over a newborn
      and mom’s chest. A hat may be added to prevent heat loss from the head, but these methods are
      supplemental to skin-to-skin care.
      CN: Health promotion and maintenance; CL: Apply
How well did you know this?
1
Not at all
2
3
4
5
Perfectly
10
Q
  1. After the nurse explains to the mother of a male neonate scheduled to receive an injection of
    vitamin K soon after birth about the rationale for the medication, which of the following statements by
    the mother indicates successful teaching?
  2. “My baby doesn’t have the normal bacteria in his intestines to produce this vitamin.”
  3. “My baby is at a high risk for a problem involving his blood’s ability to clot.”
  4. “The red blood cells my baby formed during pregnancy are destroying the vitamin K.”
  5. “My baby’s liver is not able to produce enough of this vitamin so soon after birth.
A
    1. For vitamin K synthesis in the intestines to begin, food and normal intestinal flora are
      needed. However, at birth, the neonate’s intestines are sterile. Therefore, vitamin K is administered
      via injection to prevent a vitamin K deficiency that may result in a bleeding tendency. When
      administered, vitamin K promotes formation in the liver of clotting factors II, VII, IX, and X.
      Neonates are not normally susceptible to clotting disorders, unless they are diagnosed with
      hemophilia or demonstrate a deficiency of or a problem with clotting factors. Hemolysis of fetal red
      blood cells does not destroy vitamin K. Hemolysis may be caused by Rh or ABO incompatibility,
      which leads to anemia and necessitates an exchange transfusion. Vitamin K synthesis occurs in the
      intestines, not the liver.
      CN: Pharmacological and parenteral therapies; CL: Evaluate
How well did you know this?
1
Not at all
2
3
4
5
Perfectly
11
Q
  1. The nurse is teaching the mother of a newborn to develop her baby’s sensory system. To
    further improve the infant’s most developed sense, the nurse should instruct the mother to:
  2. Speak in a high-pitched voice to get the newborn’s attention.
  3. Place the newborn about 12 inches from maternal face for best sight.
  4. Stroke the newborn’s cheek with her nipple to direct the baby’s mouth to nipple.
  5. Give infant formula with a sweetened taste to stimulate feeding.
A
    1. Currently, touch is believed to be the most highly developed sense at birth. It is probably
      why neonates respond well to touch. Auditory sense typically is relatively immature in the neonate, as
      evidenced by the neonate’s selective response to the human voice. By 4 months, the neonate should
      turn his eyes and head toward a sound coming from behind. Visual sense tends to be relativelyimmature. At birth, visual acuity is estimated at 20/100 to 20/150, but it improves rapidly during
      infancy and toddlerhood. Taste is well developed, with a preference toward glucose; however, touch
      is more developed at birth.
      CN: Health promotion and maintenance; CL: Synthesize
How well did you know this?
1
Not at all
2
3
4
5
Perfectly
12
Q
  1. The nurse has completed discharge teaching with new parents who have been discharged
    home bottle-feeding a normal term newborn. Which of the following responses indicates the need for
    further teaching?
  2. “Our baby will require feedings through the night for several weeks/months after birth.”
  3. “The baby should burp during and after each feeding with no projective vomiting.”
  4. “Our baby should have 1 to 3 soft, formed stools a day.”
  5. “We should weigh our baby daily to make sure he is gaining weight.”
A
    1. Healthy infants are weighed during their visits to their primary care provider, so it is not
      necessary to monitor weights at home. Infants may require 1 to 3 feedings during the night initially. By
      3 months, 90% of babies sleep through the night. Projective vomiting may indicate pyloric stenosis
      and should not be seen in a normal newborn. Bottle-fed infants may stool 1 to 3 times daily.
      CN: Health promotion and maintenance; CL: Evaluate
How well did you know this?
1
Not at all
2
3
4
5
Perfectly
13
Q
  1. The nurse knows the mother of a neonate has understood her car seat safety instructions when
    she comments:
  2. “I did not realize that even children between 1 to 2 years old are safer in rear-facing car
    seats.”
  3. “I should put my car seat in the front so I can watch my baby when I drive.”
  4. “I plan to use the car seat I saved from my last baby 10 years ago.”
  5. “The front-facing car seats do a better job supporting the head and neck of my baby.”
A
    1. The head and neck are best supported in a rear-facing seat in infants and toddlers, and
      infants should remain rear facing for as long possible until they outgrow their car seat. In the United
      States, the American Academy of Pediatrics recommends a rear-facing car seat for children younger
      than 2 years. The middle of the back seat is safest for a car seat. Because plastic can become brittle
      over time, car seats have an expiration date that must be checked before use. Ten years would
      generally be outside of most car seats’ expiration dates.
      CN: Health promotion and maintenance; CL: Evaluate
How well did you know this?
1
Not at all
2
3
4
5
Perfectly
14
Q
  1. While making a home visit to a primiparous client and her 3-day-old son, the nurse observes
    the mother changing the baby’s disposable diaper. Before putting the clean diaper on the neonate, the
    mother begins to apply baby powder to the neonate’s buttocks. Which of the following statements
    about baby powder should the nurse relate to the mother?
  2. It may cause pneumonia to develop.
  3. It helps prevent diaper rash.
  4. It keeps the diaper from adhering to the skin.
  5. It can result in allergies later in life.
A
    1. The nurse should inform the mother that baby powder can enter the neonate’s lungs and
      result in pneumonia secondary to aspiration of the particles. The best prevention for diaper rash is
      frequent diaper changing and keeping the neonate’s skin dry. The new disposable diapers have
      moisture-collecting materials and generally do not adhere to the skin unless the diaper becomes
      saturated. Typically, allergies are not associated with the use of baby powder in neonates.
      CN: Reduction of risk potential; CL: Synthesize
How well did you know this?
1
Not at all
2
3
4
5
Perfectly
15
Q
  1. After teaching a new mother about the care of her neonate after circumcision with a Gomco
    clamp, which of the following statements by the mother indicates to the nurse that the mother needs
    additional instructions?
  2. “The petroleum gauze may fall off into the diaper.”
  3. “A few drops of blood oozing from the site is normal.”
  4. “I’ll leave the gauze in place for 24 hours.”
  5. “I’ll remove any yellowish crusting gently with water.”
A
    1. The mother needs further instruction when she says that a yellowish crust should be
      removed with water. The yellowish crust is normal and indicates scar formation at the site. It should
      not be removed, because to do so might cause increased bleeding. The petroleum gauze prevents the
      diaper from sticking to the circumcision site, and it may fall off in the diaper. If this occurs, the
      mother should not attempt to replace it but should simply apply plain petroleum jelly to the site. The
      gauze should be left in place for 24 hours, and the mother should continue to apply petroleum jelly
      with each diaper change for 48 hours after the procedure. A few drops of oozing blood is normal, but
      if the amount is greater than a few drops the mother should apply pressure and contact the primary
      health care provider. Any bleeding after the first day should be reported.
      CN: Reduction of risk potential; CL: Evaluate
How well did you know this?
1
Not at all
2
3
4
5
Perfectly
16
Q
  1. After completing discharge instructions for a primiparous client who is bottle-feeding her
    term neonate, the nurse determines that the mother understands the instructions when the mother says
    that she should contact the primary health care provider if the neonate exhibits which of the
    following?
  2. Ability to fall asleep easily after each feeding.
  3. Spitting up of a tablespoon of formula after feeding.
  4. Passage of a liquid stool with a watery ring.
  5. Production of one to two light brown stools daily.
A
    1. The mother demonstrates understanding of the discharge instructions when she says that she
      should contact the primary health care provider if the baby has a liquid stool with a watery ring,
      because this indicates diarrhea. Infants can become dehydrated very quickly, and frequent diarrhea
      can result in dehydration. Normally, babies fall asleep easily after a feeding because they are
      satisfied and content. Spitting up a tablespoon of formula is normal. However, projectile or forceful
      vomiting in larger amounts should be reported. Bottle-fed infants typically pass one to two light
      brown stools each day.
      CN: Reduction of risk potential; CL: Evaluate
How well did you know this?
1
Not at all
2
3
4
5
Perfectly
17
Q
  1. The nurse instructs a primiparous client about bottle-feeding her neonate. Which of the
    following demonstrates that the mother has understood the nurse’s instructions?
  2. Placing the neonate on his back after the feeding.
  3. Bubbling the baby after 1 oz (30 mL) of formula.
  4. Putting three-fourths of the bottle nipple into the baby’s mouth.
  5. Pointing the nipple toward the neonate’s palate.
A
    1. Placing the neonate on his back after the feeding is recommended to minimize the risk forsudden infant death syndrome (SIDS). Placing the neonate on the abdomen after feeding has been
      associated with SIDS. The mother should bubble or burp the baby after 1⁄2 oz (15 mL) of formula has
      been taken and then again when the baby is finished. Waiting until the baby has eaten 1 oz (30 mL) of
      formula can lead to regurgitation. The entire nipple should be placed on top of the baby’s tongue and
      into the mouth to prevent excessive air from being swallowed. The nipple is pointed directly into the
      mouth, not toward the neonate’s palate, to provide adequate sucking.
      CN: Reduction of risk potential; CL: Evaluate
How well did you know this?
1
Not at all
2
3
4
5
Perfectly
18
Q
  1. The nurse is to draw a blood sample for glucose testing from a term neonate during the first
    hour after birth. The nurse should obtain the blood sample from the neonate’s foot near which of the
    following areas?
    1.
    2.
    3.
    4.
A
    1. In a neonate, the lateral aspect of the heel is the most appropriate site for obtaining a blood
      specimen. Using this area prevents damage to the calcaneus bone, which is located in the middle of
      the heel. The middle of the heel is to be avoided because of the increased risk for damaging the
      calcaneus bone located there. The middle of the foot contains the medial plantar nerve and the medial
      plantar artery, which could be injured if this site is selected. Using the base of the big toe as the site
      for specimen collection would cause a great deal of discomfort for the neonate; therefore, it is not the
      preferred site.
      CN: Reduction of risk potential; CL: Apply
How well did you know this?
1
Not at all
2
3
4
5
Perfectly
19
Q
  1. After circumcision with a Plastibell, the nurse should instruct the neonate’s mother to cleanse
    the circumcision site with which of the following?
  2. Antibacterial soap.
  3. Warm water.
  4. Povidone-iodine (Betadine) solution.
  5. Diluted hydrogen peroxide.
A
    1. After circumcision with a Plastibell, the most commonly recommended procedure is to
      clean the circumcision site with warm water with each diaper change. Other treatments are necessary
      only if complications, such as an infection, develop. Antibacterial soap or diluted hydrogen peroxide
      may cause pain and is not recommended. Povidone-iodine solution may cause stinging and burning,
      and therefore its use is not recommended.
      CN: Health promotion and maintenance; CL: Apply
How well did you know this?
1
Not at all
2
3
4
5
Perfectly
20
Q
20. Approximately 90 minutes after birth, the nurse should encourage the mother of a term
neonate to do which of the following?
1. Feed the neonate.
2. Allow the neonate to sleep.
3. Get to know the neonate.
4. Change the neonate's diaper.
A
    1. As part of the neonate’s physiologic adaptation to birth, at 90 minutes after birth the neonate
      typically is in the rest or sleep phase. During this time, the heart and respiratory rates slow and the
      neonate sleeps, unresponsive to stimuli. At this time, the mother should rest and allow the neonate to
      sleep. Feedings should be given during the first period of reactivity, considered the first 30 minutes
      after birth. During this period, the neonate’s respirations and heart rate are elevated. Getting to know
      the neonate typically occurs within the first hour after birth and then when the neonate is awake and
      during feedings. Changing the neonate’s diaper can occur at any time, but at 90 minutes after birth the
      neonate is usually in a deep sleep, unresponsive, and probably hasn’t passed any meconium.
      CN: Health promotion and maintenance; CL: Apply
How well did you know this?
1
Not at all
2
3
4
5
Perfectly
21
Q

Physical Assessment of the Neonatal Client
21. The nurse is to assess a newborn for incurving of the trunk. Which illustration indicates the
position in which the nurse should place the newborn?
1.
2.
3.
4.

A

Physical Assessment of the Neonatal Client
21. 1. When assessing the incurving of the trunk tests for automatic reflexes in the newborn, the
nurse places the infant horizontally and in a prone position with one hand, and strokes the side of the
newborn’s trunk from the shoulder to the buttocks using the other hand. If the reflex is present, the
newborn’s trunk curves toward the stimulated side. Answer 2 shows a figure for testing for a stepping
response. Answer 3 shows a figure for testing for a tonic neck reflex. Answer 4 shows a figure for
testing for the Moro (startle) reflex.
CN: Physiological adaptation; CL: Apply

How well did you know this?
1
Not at all
2
3
4
5
Perfectly
22
Q

A full-term neonate is admitted to the normal newborn nursery. The nurse notes a Moro

reflex. What should the nurse do next?
1. Call a code.
2. Identify this reflex as a normal finding.
3. Place the neonate on seizure precautions.
4. Start supplemental oxygen.

A
    1. The Moro reflex is a normal reflex of a neonate and requires no intervention. Calling a
      code, placing the neonate on seizure precautions, and starting supplemental oxygen are not necessary
      for a normally occurring reflex.
      CN: Basic care and comfort; CL: Synthesize
How well did you know this?
1
Not at all
2
3
4
5
Perfectly
23
Q
  1. After the birth of a neonate, a quick assessment is completed. The neonate is found to be
    apneic. After quickly drying and positioning the neonate, what should the nurse do next?
  2. Assign the first Apgar score.
  3. Start positive pressure ventilation.
  4. Administer oxygen.
  5. Start cardiac compressions.
A
    1. If an infant is not breathing after the initial steps of resuscitation, the next thing the nurse
      must do is begin positive pressure ventilation. Apgar scores are an evaluation of the neonate’s status
      at 1 and 5 minutes of life. Waiting to restore respirations until after assigning an Apgar score would
      be a waste of valuable time. Oxygen alone does little good if the infant is not breathing. Chest
      compressions must be accompanied by adequate oxygenation.
      CN: Physiological adaptation; CL: Synthesize
How well did you know this?
1
Not at all
2
3
4
5
Perfectly
24
Q
  1. A 6-lb, 8-oz (2,948 g) neonate was born vaginally at 38 weeks’ gestation. At 5 minutes of
    life, the neonate has the following signs: heart rate 110, intermittent grunting with respiratory rate of
    70, flaccid tone, no response to stimulus, and overall pale white in color. The Apgar score is:
  2. 2.
  3. 3.
  4. 4.
  5. 6.
A
    1. The neonate has a heart rate >100, which earns him 2 points. His respiratory rate of 70 is
      equivalent to a 2 on the scale. His flaccid muscle tone is equal to 0 on the scale. The lack of response
      to stimulus also equals 0, as does his overall pale white color. Thus, the total score equals 4.
      CN: Basic care and comfort; CL: Apply
How well did you know this?
1
Not at all
2
3
4
5
Perfectly
25
Q
25. A neonate has a large amount of secretions. After vigorously suctioning the neonate, the nurse
should assess for what possible result?
1. Bradycardia.
2. Rapid eye movement.
3. Seizures.
4. Tachycardia.
A
    1. As a result of vigorous suctioning the nurse must watch for bradycardia due to potential
      vagus nerve stimulation. Rapid eye movement is not associated with vagus nerve stimulation. Vagal
      stimulation will not cause seizures or tachycardia.
      CN: Reduction of risk potential; CL: Analyze
How well did you know this?
1
Not at all
2
3
4
5
Perfectly
26
Q
  1. When reviewing the prenatal history for a newly born neonate, the nurse notes that the mother
    has neurofibromatosis. The nurse should further assess the neonate for:
  2. Acrocyanosis.
  3. Café au lait spots.
  4. Port wine nevus.
  5. Strawberry hemangiomas.
A
    1. There is a correlation between café au lait spots and the development of neurofibromatosis.
      Acrocyanosis is a normal finding of bluish hands and feet as a result of poor capillary perfusion. Port
      wine nevus and strawberry hemangiomas are a collection of dilated capillaries and are not associated
      with any other disease process.
      CN: Reduction of risk potential; CL: Analyze
How well did you know this?
1
Not at all
2
3
4
5
Perfectly
27
Q
  1. A 24-hour-old, full-term neonate is showing signs of possible sepsis. The nurse is assisting
    the primary health care provider with a lumbar puncture on this neonate. What should the nurse do toassist in this procedure? Select all that apply.
  2. Administer the IV antibiotic.
  3. Hold the neonate steady in the correct position.
  4. Ensure a patent airway.
  5. Maintain a sterile field.
  6. Obtain a serum glucose level.
A
  1. 2,3,4. Holding the neonate steady and in the proper position will help ensure a safe and
    accurate lumbar puncture. The neonate is usually held in a “C” position to open the spaces between
    the vertebral column. This position puts the neonate at risk for airway obstruction. Thus, ensuring the
    patency of the airway is the first priority, and the nurse should observe the neonate for adequate
    ventilation. Maintaining a sterile field is important to avoid infection in the neonate. It is not
    necessary to administer antibiotics or obtain a serum glucose level during the procedure.
    CN: Safety and infection control; CL: Synthesize
How well did you know this?
1
Not at all
2
3
4
5
Perfectly
28
Q
  1. After vaginal birth of a term neonate, the nurse observes that the neonate has one artery and
    one vein in the umbilical cord. The nurse notifies the primary health care provider based on the
    analysis that this may be indicative of which anomalies?
  2. Respiratory anomalies.
  3. Musculoskeletal anomalies.
  4. Cardiovascular anomalies.
  5. Facial anomalies.
A
    1. Normally, the umbilical cord has two umbilical arteries and one vein. When a neonate is
      born with only one artery and one vein, the nurse should notify the primary health care provider for
      further evaluation of cardiac anomalies. Other common congenital problems associated with a
      missing artery include renal anomalies, central nervous system lesions, tracheoesophageal fistulas,
      trisomy 13, and trisomy 18. Respiratory anomalies are associated with dyspnea and respiratory
      distress; musculoskeletal anomalies include fractures or dislocated hip; and facial anomalies are
      associated with fetal alcohol syndrome or Down syndrome, not a missing umbilical artery.
      CN: Reduction of risk potential; CL: Analyze
How well did you know this?
1
Not at all
2
3
4
5
Perfectly
29
Q
  1. Shortly after birth, the nurse measures the circumference of a term neonate’s head and chest.
    When the two measurements are compared, which of the following would the nurse expect to find
    about the head circumference?
  2. Equal to the chest circumference.
  3. Approximately 2 cm larger than the chest.
  4. About 3 cm smaller than the chest.
  5. Approximately 4 cm larger than the chest.
A
    1. Normally at birth, the neonate’s head circumference is approximately 2 cm larger than the
      chest circumference. The average normal head circumference is 13 to 14 inches (33 to 35 cm);
      average normal chest circumference is 12.5 to 14 inches (31 to 35 cm). A head circumference that is
      equal to or smaller than the chest circumference may indicate microcephaly; a head that is larger than
      normal may indicate hydrocephalus. The presence of any of these conditions warrants further
      evaluation.
      CN: Health promotion and maintenance; CL: Analyze
How well did you know this?
1
Not at all
2
3
4
5
Perfectly
30
Q
  1. After explaining to a primiparous client about the causes of her neonate’s cranial molding,
    which of the following statements by the mother indicates the need for further instruction?
  2. “The molding was caused by an overlapping of the baby’s cranial bones during my labor.”
  3. “The amount of molding is related to the amount and length of pressure on the head.”
  4. “The molding will usually disappear in a couple of days.”
  5. “Brain damage may occur if the molding doesn’t resolve quickly.”
A
    1. The mother needs further instruction if she says the molding can result in brain damage.
      Brain damage is highly unlikely. Molding occurs during vaginal birth when the cranial bones tend to
      override or overlap as the head accommodates to the size of the mother’s birth canal. The amount and
      duration of pressure on the head influence the degree of molding. Molding usually disappears in a few
      days without any special attention.
      CN: Health promotion and maintenance; CL: Evaluate
How well did you know this?
1
Not at all
2
3
4
5
Perfectly
31
Q
  1. Which of the following observations is expected when the nurse is assessing the gestational
    age of a neonate born at term?
  2. Ear lying flat against the head.
  3. Absence of rugae in the scrotum.
  4. Sole creases covering the entire foot.
  5. Square window sign angle of 90 degrees.
A
    1. Sole creases covering the entire foot are indicative of a term neonate. If the neonate’s ear is
      lying flat against the head, the neonate is most likely preterm. An absence of rugae in the scrotum
      typically suggests a preterm neonate. A square window sign angle of 0 degrees occurs in neonates of
      40 to 42 weeks’ gestation. A 90-degree square window angle suggests an immature neonate of
      approximately 28 to 30 weeks’ gestation.
      CN: Health promotion and maintenance; CL: Apply
How well did you know this?
1
Not at all
2
3
4
5
Perfectly
32
Q
  1. While performing a complete assessment of a term neonate, which of the following findings
    would alert the nurse to notify the primary health care provider?
  2. Red reflex in the eyes.
  3. Expiratory grunt.
  4. Respiratory rate of 45 breaths/min.
  5. Prominent xiphoid process.
A
    1. An expiratory grunt is significant and should be reported promptly, because it may indicate
      respiratory distress and the need for further intervention such as oxygen or resuscitation efforts. The
      presence of a red reflex in the eyes is normal. An absent red reflex may indicate congenital cataracts.
      A respiratory rate of 45 breaths/min and a prominent xiphoid process are normal findings in a term
      neonate.
      CN: Reduction of risk potential; CL: Synthesize
How well did you know this?
1
Not at all
2
3
4
5
Perfectly
33
Q
  1. After instructing a mother about normal reflexes of term neonates, the nurse determines that
    the mother understands the instructions when she describes the tonic neck reflex as occurring when
    the neonate does which of the following?
  2. Steps briskly when held upright near a firm, hard surface.2. Pulls both arms and does not move the chin beyond the point of the elbows.
  3. Turns head to the left, extends left extremities, and flexes right extremities.
  4. Extends and abducts the arms and legs with the toes fanning open.
A
    1. The tonic neck reflex, also called the fencing position, is present when the neonate turns the
      head to the left side, extends the left extremities, and flexes the right extremities. This reflex
      disappears in a matter of months as the neonatal nervous system matures. The stepping reflex is
      demonstrated when the infant is held upright near a hard, firm surface. The prone crawl reflex is
      demonstrated when the infant pulls both arms but does not move the chin beyond the elbows. When
      the infant extends and abducts the arms and legs with the toes fanning open, this is a normal Babinski
      reflex.
      CN: Health promotion and maintenance; CL: Apply
How well did you know this?
1
Not at all
2
3
4
5
Perfectly
34
Q
  1. A primiparous client expresses concern, asking the nurse why her neonate’s eyes are crossed.
    Which of the following would the nurse include when teaching the mother about neonatal strabismus?
  2. The neonate’s eyes are unable to focus on light at this time.
  3. Neonates commonly lack eye muscle coordination.
  4. Congenital cataracts may be present.
  5. The neonate is able to fixate on distant objects immediately.
A
    1. Convergent strabismus is common during infancy until about age 6 months because of poor
      oculomotor coordination. The neonate has peripheral vision and can fixate on close objects for short
      periods. The neonate can also perceive colors, shapes, and faces. Neonates can focus on light and
      should blink or close their eyes in response to light. However, this is not associated with strabismus.
      An absent red reflex or white areas over the pupils, not strabismus, may indicate congenital cataracts.
      Most neonates cannot focus well or accommodate for distance immediately after birth.
      CN: Health promotion and maintenance; CL: Apply
How well did you know this?
1
Not at all
2
3
4
5
Perfectly
35
Q
  1. While performing a physical assessment on a term neonate shortly after birth, which of the
    following would cause the nurse to notify the primary health care provider?
  2. Deep creases across the soles of the feet.
  3. Frequent sneezing during the assessment.
  4. Single crease on each of the palms.
  5. Absence of lanugo on the skin.
A
    1. A single crease across the palm (simian crease) is most commonly associated with
      chromosomal abnormalities, notably Down syndrome. Deep creases across the soles of the feet is a
      normal finding in a term neonate. Frequent sneezing in a term neonate is normal. This occurs because
      the neonate is a nose breather and sneezing helps to clear the nares. An absence of lanugo on the skin
      of a term neonate is a normal finding.
      CN: Reduction of risk potential; CL: Synthesize
How well did you know this?
1
Not at all
2
3
4
5
Perfectly
36
Q
  1. Metabolic screening of an infant revealed a high phenylketonuria (PKU) level. Which of the
    following statements by the infant’s mother indicates understanding of the disease and its
    management? Select all that apply.
  2. “My baby can’t have milk-based formulas.”
  3. “My baby will grow out of this by the age of 2.”
  4. “This is a hereditary disease, so any future children will have it, too.”
  5. “My baby will eventually become retarded because of this disease.”
  6. “We have to follow a strict phenylalanine diet.”
  7. “A dietitian can help me plan a diet that keeps a safe phenylalanine level but lets my baby
    grow.”
A
  1. 1,5,6. Phenylketonuria, an inherited autosomal recessive disorder, involves the body’s
    inability to metabolize the amino acid phenylalanine. A diet low in phenylalanine must be followed.
    Such foods as meats, eggs, and milk are high in phenylalanine. Assistance from a dietitian is
    commonly necessary to keep phenylalanine levels low and to provide the essential amino acids
    necessary for cell function and tissue growth. With autosomal recessive disorders, future children
    will have a 25% chance of having the disease, a 50% chance of carrying the disease, and a 25%
    chance of being free of the disease. If a diet low in phenylalanine is followed until brain growth is
    complete (sometime in adolescence), the child should achieve normal intelligence.
    CN: Health promotion and maintenance; CL: Evaluate
How well did you know this?
1
Not at all
2
3
4
5
Perfectly
37
Q
  1. Assessment of a term neonate at 2 hours after birth reveals a heart rate of less than 100 bpm,
    periods of apnea approximately 25 to 30 seconds in length, and mild cyanosis around the mouth. The
    nurse notifies the primary health care provider based on the interpretation that these findings may lead
    to which condition?
  2. Respiratory arrest.
  3. Bronchial pneumonia.
  4. Intraventricular hemorrhage.
  5. Epiglottitis.
A
    1. Periods of apnea lasting longer than 20 seconds, mild cyanosis, and a heart rate of <100
      bpm (bradycardia) are associated with a potentially life-threatening event and subsequent respiratory
      arrest. The neonate needs further evaluation by the primary health care provider. Pneumonia is
      associated with tachycardia, anorexia, malaise, cyanosis, diminished breath sounds, and crackles.
      Intraventricular hemorrhage is associated with prematurity. Assessment findings include bulging
      fontanels and seizures. Epiglottitis is a bacterial form of croup. Assessment findings include
      inspiratory stridor, cough, and irritability. It occurs most commonly in children age 3 to 7 years.
      CN: Reduction of risk potential; CL: Analyze
How well did you know this?
1
Not at all
2
3
4
5
Perfectly
38
Q
  1. A new mother asks, “When will the soft spot near the front of my baby’s head close?” The
    nurse should tell the mother the soft spot will close in about:
  2. 2 to 3 months.
  3. 6 to 8 months.
  4. 9 to 10 months.
  5. 12 to 18 months.
A
    1. Normally, the anterior fontanel closes between ages 12 and 18 months. Premature closure
      (craniostenosis or premature synostosis) prevents proper growth and expansion of the brain, resulting
      in mental retardation. The posterior fontanel typically closes by ages 2 to 3 months.
      CN: Health promotion and maintenance; CL: Apply
How well did you know this?
1
Not at all
2
3
4
5
Perfectly
39
Q
  1. Which of the following assessment findings in a term neonate would cause the nurse to notify
    the primary health care provider?
  2. Absence of tears.2. Unequally sized corneas.
  3. Pupillary constriction to bright light.
  4. Red circle on pupils with ophthalmoscopic examination.
A
    1. Corneas of unequal size should be reported because this may indicate congenital glaucoma.
      An absence of tears is common because the neonate’s lacrimal glands are not yet functioning. The
      neonate’s pupils normally constrict when a bright light is focused on them. The finding implies that
      light perception and visual acuity are present, as they should be after birth. A red circle on the pupils
      is seen when an ophthalmoscope’s light shines onto the retina and is a normal finding. Called the red
      reflex, this indicates that the light is shining onto the retina.
      CN: Reduction of risk potential; CL: Synthesize
How well did you know this?
1
Not at all
2
3
4
5
Perfectly
40
Q
  1. At 24 hours of age, assessment of the neonate reveals the following: eyes closed, skin pink,
    no sign of eye movements, heart rate of 120 bpm, and respiratory rate of 35 breaths/min. The nurse
    interprets these findings as indicating that this neonate is most likely experiencing which of the
    following?
  2. Drug withdrawal.
  3. First period of reactivity.
  4. A state of deep sleep.
  5. Respiratory distress.
A
    1. At 24 hours of age, the neonate is probably in a state of deep sleep, as evidenced by the
      closed eyes, lack of eye movements, normal skin color, and normal heart rate and respiratory rate.
      Jitteriness, a high-pitched cry, and tremors are associated with drug withdrawal. The first period of
      reactivity occurs in the first 30 minutes after birth, evidenced by alertness, sucking sounds, and rapid
      heart rate and respiratory rate. There is no evidence to suggest respiratory distress because the
      neonate’s respiratory rate of 35 breaths/min is normal.
      CN: Health promotion and maintenance; CL: Analyze
How well did you know this?
1
Not at all
2
3
4
5
Perfectly
41
Q
  1. While assessing a male neonate whose mother desires him to be circumcised, the nurse
    observes that the neonate’s urinary meatus appears to be located on the ventral surface of the penis.
    The primary health care provider is notified because the nurse suspects which of the following?
  2. Phimosis.
  3. Hydrocele.
  4. Epispadias.
  5. Hypospadias.
A
    1. The condition in which the urinary meatus is located on the ventral surface of the penis,
      termed hypospadias, occurs in 1 of every 500 male infants. Circumcision is delayed until the
      condition is corrected surgically, usually between 6 and 12 months of age. Phimosis is an inability to
      retract the prepuce at an age when it should be retractable or by age 3 years. Phimosis may
      necessitate circumcision or surgical intervention. Hydrocele is a painless swelling of the scrotum that
      is common in neonates. It is not a contraindication for circumcision. Epispadias occurs when the
      urinary meatus is located on the dorsal surface of the penis. It is extremely rare and is commonly
      associated with bladder extrophy.
      CN: Reduction of risk potential; CL: Analyze
How well did you know this?
1
Not at all
2
3
4
5
Perfectly
42
Q

The Preterm Neonate
42. The nurse is discussing kangaroo care with the parents of a premature neonate. The nurse
should tell the parents that the advantages of kangaroo care include which of the following? Select all
that apply.
1. Enhanced bonding.
2. Increased IQ.
3. Improved physiologic stability.
4. Decreased length of stay in the neonatal intensive care unit.
5. Improved breast-feeding

A

The Preterm Neonate
42. 1, 3, 4, 5. Kangaroo care is skin-to-skin holding of a neonate by one of the parents. Research
has shown increased bonding, physiologic stability, decreased length of stay, and improved breast-
feeding for neonates who experience this method of holding. Research has not shown an increase in
IQ as a developmental outcome. The experience is usually limited to 1 to 2 hours, 2 to 3 times/day.
CN: Health promotion and maintenance; CL: Apply

How well did you know this?
1
Not at all
2
3
4
5
Perfectly
43
Q
  1. After a vaginal birth, a preterm neonate is to receive oxygen via mask. While administering
    the oxygen, the nurse would place the neonate in which of the following positions?
  2. Left side, with the neck slightly flexed.
  3. Back, with the head turned to the left side.
  4. Abdomen, with the head down.
  5. Back, with the neck slightly extended.
A
    1. When receiving oxygen by mask, the neonate is placed on the back with the neck slightly
      extended, in the “sniffing” or neutral position. This position optimizes lung expansion and places the
      upper respiratory tract in the best position for receiving oxygen. Placing a small rolled towel under
      the neonate’s shoulders helps to extend the neck properly without overextending it. Once stabilized
      and transferred to an isolette in the intensive care unit, the neonate can be positioned in the prone
      position, which allows for lung expansion in the oxygenated environment. Placing the neonate on the
      left side does not allow for maximum lung expansion. Also, slightly flexing the neck interferes with
      opening the airway. Placing the neonate on the back with the head turned to the left side does not
      allow for lung expansion. Placing the neonate on the abdomen interferes with proper positioning of
      the oxygen mask.
      CN: Pharmacological and parenteral therapies; CL: Synthesize
How well did you know this?
1
Not at all
2
3
4
5
Perfectly
44
Q
  1. Which of the following actions should the nurse take when performing external chest
    compressions on a neonate born at 28 weeks’ gestation?
  2. Maintain a compression to ventilation ratio of 3:1.
  3. Compress the sternum with the palm of the hand.
  4. Compress the chest 70 to 80 times/min.
  5. Displace the chest wall half the depth of the anterior-posterior diameter of the chest
A
    1. Chest compressions should be alternated with ventilation to ensure breathing and
      circulation. Two fingers or two thumbs encirciling hands, not the palm of the hand, are used to
      compress a neonate’s sternum. The chest is compressed 100 to 120 times/min. The proper technique
      recommended by the Neonatal Resuscitation Program is to use enough pressure to depress the sternum
      to a depth of approximately one-third of the anterior-posterior diameter of the chest.
      CN: Physiological adaptation; CL: Apply
How well did you know this?
1
Not at all
2
3
4
5
Perfectly
45
Q
  1. A preterm neonate who has been stabilized is placed in a radiant warmer and is receiving
    oxygen via an oxygen hood. While administering oxygen in this manner, the nurse should do which of
    the following?
  2. Humidify the air being delivered.
  3. Cover the neonate’s scalp with a warm cap.
  4. Record the neonate’s temperature every 3 to 4 minutes.
  5. Assess the neonate’s blood glucose level.
A
    1. Whenever oxygen is administered, it should be humidified to prevent drying of the nasal
      passages and mucous membranes. Because the neonate is under a radiant warmer, a stocking cap isnot necessary. Temperature, continuously monitored by a skin probe attached to the radiant warmer, is
      recorded every 30 to 60 minutes initially. Although the oxygen concentration in the hood requires
      close monitoring and measurement of blood gases, checking the blood glucose level is not necessary.
      CN: Pharmacological and parenteral therapies; CL: Apply
How well did you know this?
1
Not at all
2
3
4
5
Perfectly
46
Q
  1. Two hours ago, a neonate at 38 weeks’ gestation and weighing 3,175 g (7 lb) was born to a
    primiparous client who tested positive for beta-hemolytic Streptococcus. Which of the following
    would alert the nurse to notify the primary health care provider?
  2. Alkalosis.
  3. Increased muscle tone.
  4. Temperature instability.
  5. Positive Babinski’s reflex.
A
    1. The neonate is at high risk for sepsis due to exposure to the mother’s infection. Temperature
      instability in a neonate at 38 weeks’ gestation is an early sign of sepsis. Other signs include
      tachycardia, decreased muscle tone, acidosis, apnea, respiratory distress, hypotension, poor feeding
      behaviors, vomiting, and diarrhea. Late signs of infection include jaundice, seizures, enlarged liver
      and spleen, respiratory failure, and shock. Alkalosis is not typically seen in neonates who develop
      sepsis. Acidosis and respiratory distress may develop unless treatment such as antibiotics is started.
      A positive Babinski reflex is a normal finding and does not need to be reported.
      CN: Reduction of risk potential; CL: Analyze
How well did you know this?
1
Not at all
2
3
4
5
Perfectly
47
Q
  1. Assessment of a 2-day-old neonate born at 34 weeks’ gestation reveals absent apical pulse
    left of the midclavicular line, cyanosis, grunting, and diminished breath sounds. The priority
    intervention is to:
  2. Obtain a prescription for a stat chest x-ray.
  3. Reposition the neonate and then assess if the grunting and cyanosis resolve.
  4. Begin oxygen administration at 6 to 8 L via mask.4. Obtain a complete blood count to determine infection.
A
    1. With an absent apical pulse left of the midclavicular line accompanied by cyanosis,
      grunting, and diminished breath sounds, the neonate is most likely experiencing pneumothorax.
      Pneumothorax occurs when alveoli are overdistended and subsequently the lung collapses,
      compressing the heart and lung and compromising the venous return to the right side of the heart. This
      condition can be confirmed by x-ray or ultrasound studies. Repositioning the infant may open the
      airway, administering oxygen will improve oxygen saturation levels, and obtaining blood studies for
      infection will rule that out, but until pneumothorax is resolved, the other symptoms will continue.
      CN: Physiological adaptation; CL: Synthesize
How well did you know this?
1
Not at all
2
3
4
5
Perfectly
48
Q
  1. Twenty-four hours after cesarean birth, a neonate at 30 weeks’ gestation is diagnosed with
    respiratory distress syndrome (RDS). When explaining to the parents about the cause of this
    syndrome, the nurse should include a discussion about an alteration in the body’s secretion of which
    of the following?
  2. Somatotropin.
  3. Surfactant.
  4. Testosterone.
  5. Progesterone.
A
    1. RDS, previously called hyaline membrane disease, is a developmental condition involving
      a decrease in lung surfactant leading to improper expansion of the lung alveoli. Surfactant contains a
      group of surface-active phospholipids, of which one component—lecithin—is the most critical for
      alveolar stability. Surfactant production peaks at about 35 weeks’ gestation. This syndrome primarily
      attacks preterm neonates, although it can also affect term and postterm neonates. Altered somatotropin
      secretion is associated with growth disorders such as gigantism or dwarfism. Altered testosterone
      secretion is associated with masculinization. Altered progesterone secretion is associated with
      spontaneous abortion during pregnancy.
      CN: Physiological adaptation; CL: Apply
How well did you know this?
1
Not at all
2
3
4
5
Perfectly
49
Q
  1. A viable male neonate born to a 28-year-old multiparous client by cesarean section because
    of placenta previa is diagnosed with respiratory distress syndrome (RDS). Which of the following
    would the nurse explain as the factor placing the neonate at the greatest risk for this syndrome?
  2. Mother’s development of placenta previa.
  3. Neonate born preterm.
  4. Mother receiving analgesia 4 hours before birth.
  5. Neonate with sluggish respiratory efforts after birth.
A
    1. RDS is a developmental condition that primarily affects preterm infants before 35 weeks’
      gestation because of inadequate lung development from deficient surfactant production. The
      development of placenta previa has little correlation with the development of RDS. Although
      excessive analgesia can depress the neonate’s respiratory condition if it is given shortly before birth,
      the scenario presents no information that this has occurred. The neonate’s sluggish respiratory activity
      postpartum is not the likely cause of RDS but may be a sign that the neonate has the condition.
      CN: Reduction of risk potential; CL: Synthesize
50
Q
  1. While the nurse is caring for a neonate at 32 weeks’ gestation in an isolette with continuous
    oxygen administration, the neonate’s mother asks why the neonate’s oxygen is humidified. The nurse
    should tell the mother:
  2. “The humidity promotes expansion of the neonate’s immature lungs.”
  3. “The humidity helps to prevent viral or bacterial pneumonia.”
  4. “Oxygen is drying to the mucous membranes unless it is humidified.”
  5. “Circulation to the baby’s heart is improved with humidified oxygen.”
A
    1. Oxygen should be humidified before administration to help prevent drying of the mucous
      membranes in the respiratory tract. Drying impedes the normal functioning of cilia in the respiratory
      tract and predisposes to mucous membrane irritation. Humidification of oxygen does not promote
      expansion of the immature lungs. Expansion is promoted by placing the infant in a prone position or
      providing the preterm infant with surfactant medication. Humidified oxygen does not prevent viral or
      bacterial pneumonia. In fact, in some nurseries, Staphylococcus aureus has been detected in moistenvironments and on the hands and nails of staff members, predisposing the neonate to pneumonia.
      Humidified oxygen does not improve blood circulation in the cardiac system.
      CN: Pharmacological and parenteral therapies; CL: Apply
51
Q
  1. A preterm neonate admitted to the neonatal intensive care unit at about 30 weeks’ gestation is
    placed in an oxygenated isolette. The neonate’s mother tells the nurse that she was planning to breast-
    feed the neonate. Which of the following instructions about breast-feeding would be most
    appropriate?
  2. Breast-feeding is not recommended because the neonate needs increased fat in the diet.
  3. Once the neonate no longer needs oxygen and continuous monitoring, breast-feeding can be
    done.
  4. Breast-feeding is contraindicated because the neonate needs a high-calorie formula every 2
    hours.
  5. Gavage feedings using breast milk can be given until the neonate can coordinate sucking and
    swallowing.
A
    1. Many intensive care units that care for high-risk neonates recommend that the mother pump
      her breasts, store the milk, and bring it to the unit so the neonate can be fed with it, even if the neonate
      is being fed by gavage. As soon as the neonate has developed a coordinated suck-and-swallow
      reflex, breast-feeding can begin. Secretory immunoglobulin A, found in breast milk, is an important
      immunoglobulin that can provide immunity to the mucosal surfaces of the gastrointestinal tract. It can
      protect the neonate from enteric infections, such as those caused by Escherichia coli and Shigella
      species. Some studies have also shown that breast-fed preterm neonates maintain transcutaneous
      oxygen pressure and body temperature better than bottle-fed neonates. There is some evidence that
      breast milk can decrease the incidence of necrotizing enterocolitis. The preterm neonate does not
      need additional fat in the diet. However, some neonates may need an increased caloric intake. In such
      cases, breast milk can be fortified with an additive to provide additional calories. Neonates who are
      receiving oxygen can breast-feed. During feedings, supplemental oxygen can be delivered by nasal
      cannula.
      CN: Health promotion and maintenance; CL: Apply
52
Q
  1. Which of the following best identifies the reason for assessing a neonate weighing 1,500 g at
    32 weeks’ gestation for retinopathy of prematurity (ROP)?
  2. The neonate is at risk because of multiple factors.
  3. Oxygen is being administered at a level of 21%.
  4. The neonate was alkalotic immediately after birth.
  5. Phototherapy is likely to be prescribed by the primary health care provider.
A
    1. ROP, previously called retrolental fibroplasia, is associated with multiple risk factors,
      including high arterial blood oxygen levels, prematurity, and very low birth weight (less than 1,500
      g). In the early acute stages of ROP, the neonate’s immature retinal vessels constrict. If
      vasoconstriction is sustained, vascular closure follows, and irreversible capillary endothelial
      damage occurs. Normal room air is at 21%. Acidosis, not alkalosis, is commonly seen in preterm
      neonates, but this is not related to the development of ROP. Phototherapy is not related to the
      development of ROP. However, during phototherapy, the neonate’s eyes should be constantly covered
      to prevent damage from the lights.
      CN: Reduction of risk potential; CL: Apply
53
Q
  1. Which of the following subjects should the nurse include when teaching the mother of a
    neonate diagnosed with retinopathy of prematurity (ROP) about possible treatment for complications?1. Laser therapy.
  2. Cromolyn sodium eye drops.
  3. Frequent testing for glaucoma.
  4. Corneal transplants.
A
    1. Because the retina may become detached with ROP, laser therapy has been used
      successfully in some medical centers to treat ROP. Cromolyn sodium is used to treat seasonal
      allergies. ROP is not associated with glaucoma, so frequent testing is not necessary. Because the
      vessels of the eye are affected, not the corneas, corneal transplantation is not used.
      CN: Physiological adaptation; CL: Apply
54
Q
  1. Three days after admission of a neonate born at 30 weeks’ gestation, the neonatologist plans
    to assess the neonate for intraventricular hemorrhage (IVH). The nurse should plan to assist the
    neonatologist by preparing the neonate for which of the following?
  2. Cranial ultrasonography.
  3. Arterial blood specimen collection.
  4. Radiographs of the skull.
  5. Complete blood count specimen collection.
A
    1. Neonates who weigh less than 1,500 g or are born at less than 34 weeks’ gestation are
      susceptible to IVH. Cranial ultrasound scanning can confirm the diagnosis. The spinal fluid will show
      an increased number of red blood cells. Arterial blood gas specimen collection is done to evaluate
      the neonate’s oxygen saturation level. Skull radiographs are not commonly used because of the danger
      of radiation. Additionally, computed tomography scans have replaced the use of skull x-ray films
      because they can provide more definitive results. Complete blood count specimen collection is
      usually performed to determine the hemoglobin, hematocrit, and white blood cell count. The results
      are not specific for PIVH.
      CN: Reduction of risk potential; CL: Apply
55
Q
  1. Which of the following would the nurse most expect to assess in a neonate born at 28 weeks’
    gestation who is diagnosed with intraventricular hemorrhage (IVH)?
  2. Increased muscle tone.
  3. Hyperbilirubinemia.
  4. Bulging fontanels.
  5. Hyperactivity.
A
    1. A common finding of IVH is a bulging fontanel. The most common site of hemorrhage is the
      periventricular subependymal germinal matrix, where there is a rich blood supply and where thecapillary walls are thin and fragile. Rapid volume expansion, hypercarbia, and hypoglycemia
      contribute to the development of IVH. Other common manifestations include neurologic signs such as
      hypotonia, lethargy, temperature instability, nystagmus, apnea, bradycardia, decreased hematocrit, and
      increasing hypoxia. Seizures also may occur. Hyperbilirubinemia refers to an increase in bilirubin in
      the blood and may be seen if bleeding was severe.
      CN: Physiological adaptation; CL: Analyze
56
Q
  1. An infant born premature at 34 weeks is receiving gavage feedings. The client holding her
    infant asks why the nurse places a pacifier in the infant’s mouth during these feedings. The nurse
    replies that the pacifier helps in what ways? Select all that apply.
  2. Teaches the infant to coordinate the swallow.
  3. Provides oral stimulation.
  4. Keeps oral mucous membranes moist while the tube is in place.
  5. Reminds the infant how to suck.
  6. Stimulates secretions that help gastric emptying.
A
  1. 2,4,5. Nonnutritive sucking has been seen in infants as early as 28 weeks and ultrasound
    examinations have shown thumb sucking in utero even earlier. Nonnutritive sucking provides oral
    stimulation and allows the baby to maintain the sucking reflex needed for breast- or bottle-feedings
    later. It does not teach the infant how to suck and swallow. Sucking is thought to help with gastric
    emptying by stimulating secretions of GI peptides. Moisture of the mucous membranes is an indication
    of adequate hydration, and nonnutritive sucking will not have an effect.
    CN: Basic care and comfort; CL: Apply
57
Q
  1. While caring for a neonate born at 32 weeks’ gestation, the nurse assesses the neonate daily
    for symptoms of necrotizing enterocolitis (NEC). Which of the following would alert the nurse to
    notify the neonatologist?
  2. The presence of 1 mL of gastric residual before a gavage feeding.
  3. Jaundice appearing on the face and chest.
  4. An increase in bowel peristalsis.
  5. Abdominal distention.
A
    1. Indications of NEC include abdominal distention with gastric retention and vomiting. Other
      signs may include lethargy, irritability, positive blood culture in stool, absent or diminished bowel
      sounds, apnea, diarrhea, metabolic acidosis, and unstable temperature. A gastric residual of 1 mL is
      not significant. Jaundice of the face and chest is associated with the neonate’s immature liver function
      and increased bilirubin, not NEC. Typically with NEC, the neonate would exhibit absent or
      diminished bowel sounds, not increased peristalsis.
      CN: Physiological adaptation; CL: Analyze
58
Q
  1. Which of the following statements by the mother of a neonate diagnosed with
    bronchopulmonary dysplasia (BPD) indicates effective teaching?
  2. “BPD is an acute disease that can be treated with antibiotics.”
  3. “My baby may require long-term respiratory support.”
  4. “Bronchodilators can cure my baby’s condition.”
  5. “My baby may have seizures later on in life because of this condition.”
A
    1. BPD is a chronic illness that may require prolonged hospitalization and permanent assisted
      ventilation. The disease typically occurs in compromised very-low-birth-weight neonates who
      require oxygen therapy and assisted ventilation for treatment of respiratory distress syndrome. The
      cause is multifactorial, and the disease has four stages. The neonate’s activities may be limited by the
      disease. Antibiotics may be prescribed, and bronchodilators may be used, but these medications will
      not cure the chronic disease state. Seizure activity is associated with periventricular-intraventricular
      hemorrhage, not BPD.
      CN: Physiological adaptation; CL: Evaluate
59
Q
  1. A preterm infant born 2 hours ago at 34 weeks’ gestation is experiencing rapid respirations,
    grunting, no breath sounds on one side, and a shift in location of heart sounds. The nurse should
    prepare to assist with which of the following?
  2. Placement of the neonate on a ventilator.2. Administration of bronchodilators through the nares.
  3. Suctioning of the neonate’s nares with wall suction.
  4. Insertion of a chest tube into the neonate.
A
    1. The client data support the diagnosis of pneumothorax, which would be confirmed with a
      chest x-ray. Pneumothorax is an accumulation of air in the thoracic cavity between the parietal and
      visceral pleurae and requires immediate removal of the accumulated air. Resolution is initiated with
      insertion of a chest tube connected to continuous negative pressure. The neonate does not need to be
      placed on a ventilator unless there is evidence of severe respiratory distress. The goal of treatment is
      to re-inflate the collapsed lung. Administering bronchodilators through the nares or suctioning the
      neonate’s nares would do nothing to aid in lung re-inflation.
      CN: Physiological adaptation; CL: Synthesize
60
Q
  1. Which of the following would alert the nurse to suspect that a neonate born at 34 weeks’
    gestation who is currently in an isolette with humidified oxygen and receiving intravenous fluids has
    developed overhydration?
  2. Hypernatremia.
  3. Polycythemia.
  4. Hypoproteinemia.
  5. Increased urine specific gravity.
A
    1. Decreased protein or hypoproteinemia is a sign of overhydration, which can lead to patent
      ductus arteriosus or congestive heart failure. Bulging fontanels, decreased serum sodium, decreased
      urine specific gravity, and decreased hematocrit are other signs of overhydration. Hypernatremia
      (increased serum sodium concentration) or increased urine specific gravity would suggest
      dehydration, not overhydration. Polycythemia evidenced by an elevated hematocrit would suggest
      hypoxia or congenital heart disorder.CN: Reduction of risk potential; CL: Analyze
61
Q
  1. A newborn weighing 61⁄2 lb (2,950 g) is to be given naloxone hydrochloride (Narcan) due to
    respiratory depression as a result of a narcotic given to the mother shortly before birth. The drug is to
    be given 0.01 mg/kg into the umbilical vein. The vial is marked 0.4 mg/mL. How many milligrams
    would the newborn receive? Round off to two decimals.
    __________________ mg.
A
  1. 0.03 mg

2,950 = 2.95 KG
2.95 KG X 0.01 MG = 0.029 MG OR 0.03 MG

CN: Pharmacological and parenteral therapies; CL: Apply

62
Q

The Postterm Neonate
62. A neonate born by cesarean at 42 weeks’ gestation, weighing 4.1 kg (9 lb, 1 oz), with Apgar
scores of 8 at 1 minute and 9 at 5 minutes after birth, develops an increased respiratory rate and
tremors of the hands and feet 2 hours postpartum. Which of the following is the priority problem?
1. Ineffective airway clearance.
2. Hyperthermia.
3. Decreased cardiac output.
4. Hypoglycemia.

A

The Postterm Neonate
62. 4. Increased respiratory rate and tremors are indicative of hypoglycemia, which commonly
affects the postterm neonate because of depleted glycogen stores. There is no indication that the
neonate has ineffective airway clearance, which would be evidenced by excessive amounts of mucus
or visualization of meconium on the vocal cords. Lethargy, not tremors, would suggest infection or
hyperthermia. Furthermore, the postterm neonate typically has difficulty maintaining temperature,
resulting in hypothermia, not hyperthermia. Decreased cardiac output is not indicated, particularly
because the neonate was born by cesarean section, which is not considered a difficult birth.
CN: Health promotion and maintenance; CL: Analyze

63
Q
  1. At a home visit, the nurse assesses a neonate born vaginally at 41 weeks’ gestation 5 days
    ago. Which of these findings warrants further assessment?
  2. Frequent hiccups.
  3. Loose, watery stool in diaper.
  4. Pink papular vesicles on the face.
  5. Dry, peeling skin.
A
    1. A loose, watery stool in the diaper is indicative of diarrhea and needs immediate attention.
      The infant may become severely dehydrated quickly because of the higher percentage of water content
      per body weight in the neonate, compared with the adult. Frequent hiccups are considered normal in a
      neonate and do not warrant additional investigation. Pink papular vesicles (erythema toxicum) on the
      face are considered normal in a neonate and disappear without treatment. Dry, peeling skin is normal
      in a postterm neonate.
      CN: Health promotion and maintenance; CL: Analyze
64
Q
  1. When performing an initial assessment of a postterm male neonate weighing 4,000 g (9 lb)
    who was admitted to the observation nursery after a vaginal birth with low forceps, the nurse detects
    Ortolani’s sign. Which of the following actions should the nurse do next?
  2. Determine the length of the mother’s labor.
  3. Notify the primary health care provider immediately.
  4. Keep the neonate under the radiant warmer for 2 hours.
  5. Obtain a blood sample to check for hypoglycemia.
A
    1. Ortolani maneuver involves flexing the neonate’s knees and hips at right angles and bringing
      the sides of the knees down to the surface of the examining table. A characteristic click or “clunk,”
      felt or heard, represents a positive Ortolani sign, suggesting a possible hip dislocation. The nurse
      should notify the primary health care provider promptly because treatment is needed, while
      maintaining the dislocated hip in a position of flexion and abduction. It should be noted that many
      institutions now limit performing the Ortolani’s maneuver to APNs or physicians. Determining the
      length of the mother’s labor provides no useful information related to the nurse’s finding. Keeping the
      infant under the radiant warmer is necessary only if the neonate’s temperature is low or unstable.
      Checking for hypoglycemia is not indicated at this time, unless the neonate is exhibiting jitteriness.
      CN: Reduction of risk potential; CL: Synthesize
65
Q
  1. A neonate is admitted to the neonatal intensive care unit for observation with a diagnosis of
    probable meconium aspiration syndrome (MAS). The neonate weighs 10 lb, 4 oz (4,650 g) and is at
    41 weeks’ gestation. Which of the following would be the priority problem?
  2. Impaired skin integrity.
  3. Hyperglycemia.
  4. Risk for impaired parent-infant-child attachment.
  5. Impaired gas exchange.
A
    1. The priority problem for the neonate with probable MAS is impaired gas exchange related
      to the effects of respiratory distress. Obstruction of the airways may be complete or partial.
      Meconium aspiration may lead to pneumonia or pneumothorax. Establishing adequate respirations is
      the primary goal. Impaired skin integrity is a concern, but establishing and maintaining an airway and
      gas exchange is always the priority. Hypoglycemia tends to be a problem for large-for-gestational-age
      babies, not hyperglycemia. If the parents do not express interest or concern for the neonate, then risk
      for impaired parent-infant-child attachment may be appropriate once the airway is established.
      CN: Physiological adaptation; CL: Analyze
66
Q
  1. When developing the initial plan of care for a neonate who was born at 41 weeks’ gestation
    with meconium aspiration syndrome (MAS) requiring mechanical ventilation, which of the following
    should the nurse include?
  2. Care of an umbilical arterial line.
  3. Frequent ultrasound scans.
  4. Orogastric feedings as soon as possible.
  5. Assessment for symptoms of hyperglycemia.
A
    1. Care of an umbilical arterial line would be included in the neonate’s plan of care because
      an umbilical arterial line is commonly inserted to monitor arterial blood pressures, blood pH, bloodgases, and infusion of intravenous fluids, blood, or medications. Frequent ultrasound scans are not
      indicated at this time. However, chest radiographs may be used to detect lung densities, because
      pneumonia is a major complication of this disorder. Orogastric feeding may not be feasible while the
      health care team focuses on interventions to establish adequate oxygenation. The neonate with MAS
      commonly experiences hypoglycemia, not hyperglycemia. Hypoglycemia occurs because of depletion
      of glucose stores related to hypothermia.
      CN: Physiological adaptation; CL: Synthesize
67
Q
  1. The neonate in the nurse’s care has a pneumothorax. The nurse knows the signs of early
    decompensation and to carefully assess which of the following? Select all that apply.
  2. B/P.
  3. Temperature.
  4. Urinary output.
  5. Color.5. Heart rate.
A
  1. 1,4,5. The pneumothorax may affect cardiac output, thus affecting perfusion causing a
    decrease in B/P and color. As the neonate attempts to compensate, bradycardia or tachycardia may be
    exhibited. A change in temperature and urinary output are very late signs of decompensation.
    CN: Physiological adaptation; CL: Analyze
68
Q

The Neonate with Risk Factors
68. A nurse is attempting to resuscitate a neonate. Thirty seconds of chest compressions have
been completed. The neonate’s heart rate remains less than 60 bpm. Epinephrine is given. What is the
expected outcome for a neonate who has received epinephrine during resuscitation?
1. Increased urine output.
2. A normal heart rate.
3. Pain relief.
4. Sedation

A

The Neonate with Risk Factors
68. 2. Epinephrine is given for severe bradycardia and hypotension. An expected outcome would
be an increased heart rate to a normal range. Epinephrine decreases renal blood flow, so a decrease
in urine output would be expected. Epinephrine also stimulates alpha- and beta-adrenergic receptors,
which do not offer pain relief or sedation.
CN: Pharmacological and parenteral therapies; CL: Evaluate

69
Q
  1. A mother is visiting her neonate in the neonatal intensive care unit. Her baby is fussy and the
    mother wants to know what to do. In order to quiet a sick neonate, which of the following can the
    nurse teach the mother to do?
  2. Bring in toys for distraction.
  3. Place a musical mobile over the crib.
  4. Stroke the neonate’s back.
  5. Use constant, gentle touch.
A
    1. Neonates that are sick do not have the physical resources or energy to respond to all
      elements of the environment. The use of a constant touch provides comfort and only requires one
      response to a stimulus. To comfort a sick neonate, the care provider applies gentle, constant physical
      support or touch. Toys for distraction are not developmentally appropriate for a neonate. Sick
      neonates react to any stimulus; in responding, the sick neonate may have increased energy demands
      and increased oxygen requirements. A musical mobile may be too much audio stimulation and thus
      increases energy and oxygen demands. Repetitive touching with a hand going off and on the neonate,
      as with stroking or patting, requires the neonate to respond to every touch, thus increasing energy and
      oxygen demands.
      CN: Basic care and comfort; CL: Synthesize
70
Q
  1. A neonate born at 40 weeks’ gestation admitted to the nursery is found to be hypoglycemic. At
    4 hours of age, the neonate appears pale and his pulse oximeter is reading 75%. The nurse should:
  2. Increase the IV rate.
  3. Provide supplemental oxygen.
  4. Record the finding on the chart and repeat the reading in 30 minutes.
  5. Wrap the neonate to increase body temperature.
A
    1. Recommended pulse oximetry reading in a full-term neonate is 95% to 100%. The
      saturation reading of only 75% is an indication that the neonate is not adequately oxygenating in room
      air. Providing supplemental oxygen will increase the neonate’s oxygen saturation. Increasing the IV
      rate will not improve the oxygen saturation. Documenting the finding and taking no action is not
      appropriate with a saturation of 75%. Wrapping and increasing the body temperature of the neonate
      may increase the saturation reading only if it is inaccurate due to cold extremities. Caution must be
      used because overheating a neonate can be harmful.
      CN: Reduction of risk potential; CL: Synthesize
71
Q
  1. A neonate born at 29 weeks’ gestation received nasal continuous positive airway pressure.
    The neonate is receiving oxygen at 1 L/min via nasal cannula at a fraction of inspired oxygen (F IO2 ) of
    0.23. The pulse oximetry reading is 70% saturation. In which order of priority from first to last
    should the nurse take these actions?
  2. Increase the F IO2 .
  3. Make sure the pulse oximeter is correlating to the heart rate.
  4. Assess the neonate for color.
  5. Assess the neonate for respiratory effort.
A

71.
4. Assess the neonate for respiratory effort.
3. Assess the neonate for color.2. Make sure the pulse oximeter is correlating to the heart rate.
1. Increase the FIO2 .
Assessment of the neonate is the most important priority and should be completed first.
Respiratory effort must be assessed first to determine if the neonate is breathing. Once breathing is
established, assessment of color is next. Then verification of proper equipment function is necessary.
The last measure is to increase the F IO2 due to the potential harmful effects of oxygen. Oxygen should
be viewed as a drug and its use evaluated carefully. Increasing the F IO2 should be done only when
indicated; possible causes of low saturation readings must be evaluated first.
CN: Reduction of risk potential; CL: Synthesize

72
Q
  1. A neonate with heart failure is being discharged home. In teaching the parents about the
    neonate’s nutritional needs, the nurse should explain that:
  2. Fluids should be restricted.
  3. Decreased activity level should reduce the need for additional calories.
  4. The formula should be low in sodium.
  5. The neonate may need a more calorie-dense formula.
A
    1. Neonates with heart failure may need calorie-dense formula to provide extra calories for
      growth. Fluids should not be restricted because the nutritional requirements are based on calories per
      ounce of formula. Decreasing fluid intake will decrease calories needed for growth. These neonates
      may have limited energy due to their heart condition but have a high caloric need to stimulate proper
      growth and development. The sodium level should be at a normal level to ensure adequate fluid and
      electrolyte balance unless prescribed by the primary health care provider.
      CN: Health promotion and maintenance; CL: Create
73
Q
  1. During an assessment of a neonate born at 33 weeks’ gestation, a nurse finds and reports a
    heart murmur. An echocardiogram reveals patent ductus arteriosis, for which the neonate received
    indomethacin. An expected outcome after the administration of indomethacin to a neonate with patent
    ductus arteriosis is:
  2. Closure of a patent ductus arteriosus.
  3. Decreased bleeding time.
  4. Increased gastrointestinal function.
  5. Increased renal output.
A
    1. The indication for the use of indomethacin is to close a patent ductus arteriosus. Adverse
      effects include decreased renal blood flow, platelet dysfunction with coagulation defects, decreased
      GI motility, and an increase in necrotizing enterocolitis. Thus, increased bleeding time, decreased
      gastrointestinal function, and decreased renal output would be expected outcomes after the
      administration of indomethacin.
      CN: Pharmacological and parenteral therapies; CL: Evaluate
74
Q
  1. A preterm neonate is unable to breast- or bottle-feed. The primary health care provider
    writes a prescription to feed the neonate via nasogastric (NG) tube. When choosing an NG feeding
    tube for a neonate, the nurse should base the tube size on the neonate’s:
  2. Disease process.
  3. Gestational age.
  4. Length.
  5. Weight.
A
    1. The size of the NG feeding tube is based on the neonate’s weight. A larger feeding tube can
      be inserted into a heavier neonate. The disease process plays no role in the size of the feeding tube
      used. The neonate’s weight and size can vary widely; thus, there is no standard tube size for any
      gestational age. Length will determine the depth at which the NG tube is placed in the stomach, not the
      size of the tube.
      CN: Basic care and comfort; CL: Synthesize
75
Q
  1. A nurse is reviewing a client’s maternal prenatal record and notes that the mother used
    narcotics during her pregnancy. A primary nursing intervention when caring for a drug-exposed
    neonate is to:
  2. Assess vital signs including blood pressure every hour.
  3. Minimize environmental stimuli.
  4. Place the infant in a well-lighted area for observation.
  5. Provide stimulation to increase adaptation to the environment.
A
    1. A quiet environment with decreased stimulation is the best treatment for a drug-exposed
      neonate. The drug-exposed neonate has limited ability to deal with stress and cope with stimuli.
      Assessing vital signs with blood pressure every hour will disturb the neonate’s rest periods and cause
      increased physical and psychological demands. Placement in a well-lighted or stimulating
      environment is overwhelming for the neonate and will increase the neonate’s stress level.
      CN: Physiological adaptation; CL: Apply
76
Q
  1. The nurse is receiving over the telephone a laboratory results report of a neonate’s blood
    glucose level. The nurse should:
  2. Write down the results, read back the results to the caller from the laboratory, and receive
    confirmation from the caller that the nurse understands the results.
  3. Repeat the results to the caller from the laboratory, write the results on scrap paper first, and
    then transfer the results to the chart.
  4. Indicate to the caller that the nurse cannot receive verbal results from laboratory tests for
    neonates, and ask the laboratory to bring the written results to the nursery.
  5. Request that the laboratory send the results by e-mail to transfer to the client’s electronic
    record.
A
    1. To ensure client safety, the nurse should first write the results on the chart, then read them
      back to the caller and wait for the caller to confirm that the nurse has understood the results. Using
      scrap paper increases the risk of losing the results as well as transcription errors. The nurse mayreceive results by telephone, and while electronic transfer to the client’s electronic record is
      appropriate, the nurse can also accept the telephone results if the laboratory has called the results to
      the nursery. Sending client information via e-mail is unacceptable due to potential security and
      privacy issues.
      CN: Safety and infection control; CL: Apply
77
Q
  1. A multiparous client who has a neonate diagnosed with hemolytic disease of the newborn
    asks the nurse why the neonate has developed this problem. Which of the following responses by the
    nurse would be most appropriate?1. “You are Rh positive and the baby is Rh negative.”
  2. “You and the baby are both Rh negative.”
  3. “You are Rh negative and the baby is Rh positive.”
  4. “The baby and you are both Rh positive.”
A
    1. Hemolytic disease of the newborn is associated with Rh problems. Hemolytic disease of
      the newborn occurs most commonly when the mother is Rh-negative the infant is Rh-negative. About
      13% of Caucasians, 7% to 8% of people of African descent, and 1% of people of Asian descent are
      Rh-negative. Rh-positive cells enter the mother’s Rh-negative bloodstream, and antibodies to the Rh-
      positive cells are produced. In a subsequent pregnancy, the antibodies cross the placenta to the Rh-
      positive fetus and begin the destruction of Rh-positive cells through hemolysis. This results in severe
      fetal anemia.
      CN: Physiological adaptation; CL: Apply
78
Q
  1. After teaching the multiparous mother about hemolytic disease of the newborn and Rh
    sensitization, the nurse determines that the client understands why she was not sensitized during her
    other pregnancy when she says which of the following?
  2. “My other baby had a different father.”
  3. “Like most women, I have immunity against the Rh factor.”
  4. “Antibodies are not usually formed until after exposure to an antigen.”
  5. “My blood couldn’t neutralize antibodies formed from my first pregnancy.”
A
    1. The problem of Rh sensitivity arises when the mother’s blood develops antibodies after
      fetal red blood cells enter the maternal circulation. In cases of Rh sensitivity, this usually does not
      occur until after the first pregnancy. Hence, hemolytic disease of the newborn is rare in a primiparous
      client. A mismatched blood transfusion in the past or an unrecognized spontaneous abortion could
      also result in hemolytic disease because the transfusion or abortion would have the same effects on
      the client. The statement about the other baby having a different father may be true. However, if both
      fathers were Rh-positive, then sensitization could occur. Most women do not have immunity against
      the antibodies formed when Rh-positive cells enter the mother’s bloodstream. Antibodies are not
      neutralized by the mother’s system.
      CN: Reduction of risk potential; CL: Evaluate
79
Q
  1. After teaching a multiparous client about the effects of hemolysis due to Rh sensitization on
    the neonate at birth, the nurse determines that the client needs further instruction when the mother
    reports that the neonate may have which of the following?
  2. Cardiac decompensation.
  3. Polycythemia.
  4. Anemia.
  5. Splenic enlargement.
A
    1. The Rh-sensitized neonate generally does not have problems related to polycythemia.
      Therefore, the client needs additional teaching. In general, moderate to severe Rh sensitization can
      cause anemia, enlarged spleen, and cardiac decompensation. Cardiac decompensation (as in heart
      failure) occurs because of severe anemia. Anemia is caused by the destruction of red blood cells by
      antibodies as the severity of hemolytic disease of the neonate increases. Splenic enlargement is
      caused by the excessive destruction of fetal red blood cells.
      CN: Physiological adaptation; CL: Evaluate
80
Q
  1. After birth, a direct Coombs test is performed on the umbilical cord blood of a neonate with
    Rh-positive blood born to a mother with Rh-negative blood. The nurse explains to the client that this
    test is done to detect which of the following?
  2. Degree of anemia in the neonate.
  3. Electrolyte imbalances in the neonate.
  4. Antibodies coating the neonate’s red blood cells.
  5. Antigens coating the neonate’s red blood cells.
A
    1. A direct Coombs test is done on umbilical cord blood to detect antibodies coating the
      neonate’s red blood cells. Hematocrit is used to detect anemia. Sodium, potassium, and chloride are
      used to detect electrolyte imbalances. Antigens on the neonate’s red blood cells are proteins that help
      determine the neonate’s blood type.
      CN: Reduction of risk potential; CL: Apply
81
Q
  1. After teaching the mother of a neonate with erythroblastosis fetalis who is to receive an
    exchange transfusion, which of the following, if stated by the mother as the purpose of the transfusion,
    indicates effective teaching?
  2. To replenish the neonate’s leukocytes.
  3. To restore the fluid and electrolyte balance.
  4. To correct the neonate’s anemia.
  5. To replace Rh-negative blood with Rh-positive blood.
A
    1. An exchange transfusion is done to reduce the blood concentration of bilirubin and correct
      the anemia. The exchange transfusion does not replenish the white blood cells or restore the fluid and
      electrolyte balance. The neonate’s Rh-positive blood is replaced by Rh-negative blood.
      CN: Reduction of risk potential; CL: Apply
82
Q
  1. The nurse explains to the mother of a neonate diagnosed with erythroblastosis fetalis that the
    exchange transfusion is necessary to prevent damage primarily to which of the following organs in the
    neonate?
  2. Kidneys.
  3. Brain.
  4. Lungs.
  5. Liver.
A
    1. The organ most susceptible to damage from uncontrolled hemolytic disease is the brain.
      Bilirubin levels increase as the red blood cells are destroyed. Bilirubin crosses the blood-brainbarrier and damages the cells of the central nervous system. This condition, called kernicterus, is
      potentially fatal. Although the kidneys, lungs, and liver may be affected by increased bilirubin levels,
      the brain will sustain the most life-threatening injury.
      CN: Reduction of risk potential; CL: Apply
83
Q
  1. The nurse determines that a newborn is hypoglycemic based on which of the following
    findings? Select all that apply.
  2. A blood glucose reading of less than 30 mg/dL (1.7 mmol/L) at 1 hour.2. Family history of insulin-dependent diabetes.
  3. Internal fetal monitor tracing.
  4. Irregular respirations, tremors, and hypothermia.
  5. Large for gestational age.
A
  1. 1,4. A blood glucose reading at or below 30 mg/dL (1.7 mmol/L) within 2 hours of birth and
    irregular respirations, tremors, and hypothermia are indicative of hypoglycemia. Blood glucose
    should be 45 mg/dL (2.5 mmol/L) by 24 hours of age. Internal fetal monitors detect the strength of
    contractions and the fetal heart rate. An infant of an insulin-dependent mother and a large-for-
    gestational-age infant are at greater risk of developing hypoglycemia and need to be observed
    carefully but these findings are not definitive for the diagnosis of hypoglycemia.
    CN: Physiological adaptation; CL: Analyze
84
Q
  1. The nurse is caring for an infant of a primiparous woman with insulin-dependent diabetes.
    When the mother visits the neonate at 1 hour after birth, the nurse explains to the mother that the
    neonate is being closely monitored for symptoms of hypoglycemia because of which of the following?
  2. Increased use of glucose stores during a difficult labor and birth process.
  3. Interrupted supply of maternal glucose and continued high neonatal insulin production.
  4. A normal response that occurs during transition from intrauterine to extrauterine life.
  5. Increased pancreatic enzyme production caused by decreased glucose stores.
A
    1. Glucose crosses the placenta, but insulin does not. Hence, a high maternal blood glucose
      level causes a high fetal blood glucose level. This causes the fetal pancreas to secrete more insulin.
      At birth, the neonate loses the maternal glucose source but continues to produce much insulin, which
      commonly causes a drop in blood glucose levels (hypoglycemia), usually at 30 to 60 minutes
      postpartum. Most neonates do not develop hypoglycemia if their mothers are not insulin dependent
      unless they are preterm. Therefore, hypoglycemia is not a normal response as the neonate transitions
      to extrauterine life.
      CN: Reduction of risk potential; CL: Analyze
85
Q
  1. When caring for a neonate weighing 4,564 g (10 lb, 1 oz) born vaginally to a of a mother with
    diabetes, the nurse should assess the neonate for fracture of the:
  2. Clavicle.
  3. Skull.
  4. Wrist.
  5. Rib cage.
A
    1. Infants born to mothers with diabetes tend to be larger than average, and this neonate
      weighs 10 lb, 1 oz (4,564 g). The most common fractures are those of the clavicle and long bones,
      such as the femur. In a neonate, the skull bones are not fused and move to allow for vaginal birth, so
      skull fracture is rarely seen. Wrist and rib cages are rarely fractured.
      CN: Reduction of risk potential; CL: Apply
86
Q
  1. While caring for a neonate of a woman with diabetes soon after birth, the nurse has fed the
    newborn formula to prevent hypoglycemia. The nurse checks the neonate’s blood glucose level and it
    is 60 mg/dL (3.3 mmol/L), but the neonate continues to exhibit jitteriness and tremors. The nurse
    should first:
  2. Request a prescription for a blood calcium level.
  3. Administer glucose intravenously based on infant glucose level.
  4. Take the neonate’s temperature and place him in the radiant warmer.
  5. Refeed the infant to continue to increase the blood glucose level.
A
    1. This neonate has a mother with diabetes who tends to have higher calcium levels, which
      can cause secondary hypoparathyroidism in their neonates. This lack of calcium may be the cause of
      the tremors and jitteriness of this neonate and a serum calcium level should be obtained. Other factors
      contributing to hypocalcemia in neonates include hypophosphatemia from tissue metabolism, vitamin
      D antagonism from increased cortisol levels, and decreased serum magnesium levels. Beginning a
      glucose IV based on a normal infant glucose level would have no benefit. Rechecking the neonate’s
      temperature is a precaution that can be taken to assure that it is within normal limits but is not the
      action to take first. Refeeding the infant who has a normal newborn blood glucose level is not
      appropriate.
      CN: Management of care; CL: Synthesize
87
Q
  1. The nurse is caring for a neonate weighing 4,536 g (10 lb) who was born via cesarean
    section 1 hour ago. The mother is a class B primipara with insulin-dependent diabetes. She asks the
    nurse, “Why is my baby in the neonatal intensive care unit?” The nurse bases a response on the
    understanding that neonates of mothers with class B diabetes commonly develop which of the
    following conditions?
  2. Anemia.
  3. Persistent pulmonary hypertension.
  4. Hemolytic disease.
  5. Hypoglycemia.
A
    1. Hypoglycemia is caused by the rapid depletion of glucose stores. In addition, neonates born
      to class B diabetic women are about seven times more likely to suffer from respiratory distress
      syndrome than neonates born to nondiabetic women. This neonate should be closely monitored for
      symptoms of hypoglycemia and respiratory distress. Neonates of diabetic mothers commonly have
      polycythemia, not anemia. Anemia and hemolytic disease are associated with erythroblastosis fetalis.
      Persistent pulmonary hypertension is associated with meconium aspiration syndrome.
      CN: Reduction of risk potential; CL: Apply
88
Q
  1. While assessing a neonate weighing 3,175 g (7 lb) who was born at 39 weeks’ gestation to a
    primiparous client who admits to opiate use during pregnancy, which of the following would alert the
    nurse to possible opiate withdrawal?
  2. Bradycardia.
  3. High-pitched cry.
  4. Sluggishness.
  5. Hypocalcemia.
A
    1. Manifestations of opiate withdrawal in the neonate include an increased central nervous
      system irritability, such as a shrill, high-pitched cry, gastrointestinal symptoms, and metabolic,
      vasomotor, and respiratory disturbances. These signs usually appear within 72 hours and persist for
      several days. These neonates are difficult to console, have poor feeding behaviors, and have
      diarrhea. Bradycardia is associated with preterm neonates. Sluggishness and lethargy are associated
      with neonates whose mothers received analgesia shortly before birth. Hypocalcemia occurs most
      commonly in infants of mothers with diabetes, premature infants, and low-birth-weight infants.
      CN: Reduction of risk potential; CL: Analyze
89
Q
89. After teaching a primiparous client who used opiates during pregnancy about possiblegastrointestinal signs and symptoms in her neonate, which of the following, if stated by the mother as
common, indicates effective teaching?
1. Hypotonia.
2. Constipation.
3. Vomiting.
4. Abdominal distention.
A
    1. Neonates experiencing opiate withdrawal have gastrointestinal problems similar to those of
      adults withdrawing from opiates. The neonates exhibit poor sucking, vomiting, drooling, diarrhea,
      regurgitation, and anorexia. In addition, they are difficult to console and difficult to feed. Because of
      these problems, the neonate withdrawing from opiates needs to be monitored carefully to prevent
      dehydration. Neonates with opiate exposure experience hypertonia, not hypotonia, due to increased
      central nervous system irritability. Diarrhea, not constipation, is seen in these neonates. Abdominal
      distention is associated with necrotizing enterocolitis, not opiate withdrawal.
      CN: Reduction of risk potential; CL: Evaluate
90
Q
  1. When teaching a primiparous client who used cocaine during pregnancy how to comfort her
    fussy neonate, the nurse can advise the mother to:
  2. Tightly swaddle the neonate.
  3. Feed the neonate extra, high-calorie formula.
  4. Keep the neonate in a brightly lit environment.
  5. Touch the baby only when he is crying
A
    1. A neonate undergoing cocaine withdrawal is irritable, often restless, difficult to console,
      and often in need of increased activity. It is commonly helpful to swaddle the neonate tightly with a
      blanket, offer a pacifier, and cuddle and rock the neonate. Offering extra nourishment is not advised
      because overfeeding tends to increase gastrointestinal problems such as vomiting, regurgitation, and
      diarrhea. Environmental stimuli such as bright lights and loud noises should be kept to a minimum to
      decrease agitation. Minimizing touching of the neonate to only when he or she is crying will not aid
      the bonding process between mother and neonate. Frequent holding and touching are permissible.
      CN: Reduction of risk potential; CL: Synthesize
91
Q
  1. A neonate born at 38 weeks’ gestation is admitted to the neonatal nursery for observation. The
    neonate’s mother, who is positive for human immunodeficiency virus (HIV) infection, has received no
    prenatal care. The mother asks the nurse if her neonate is positive for HIV. The nurse can tell the
    mother which of the following?
  2. “More than 50% of neonates born to mothers who are positive for HIV will be positive at 18
    months of age.”
  3. “An enlarged liver at birth generally means the neonate is HIV positive.”
  4. “A complete blood count analysis is the primary method for determining whether the neonate is
    HIV positive.”
  5. “We will test your baby at 2 weeks of age because testing for HIV now may give a false-
    positive result.”
A
    1. Although most neonates are asymptomatic, they do test positive for HIV at birth because of
      the mother’s antibodies. It may take several months before an accurate diagnosis can be made. New
      guidelines suggest that infants should be tested at 2 to 3 weeks, 1 to 2 months, and again 4 to 6
      months. It is estimated that 15% to 30% of all HIV-positive mothers without treatment will give birth
      to HIV-positive infants. With appropriate drug intervention to the mother during pregnancy, 95% of
      these neonates can be born unaffected. An enlarged liver at birth is associated with erythroblastosis
      fetalis, not HIV infection. Virologic testing, such as deoxyribonucleic acid polymerase chain reaction,
      viral culture, or ribonucleic acid plasma assay, can diagnose HIV infection by 6 months of age and
      commonly in the first month.
      CN: Reduction of risk potential; CL: Apply
92
Q
  1. When caring for a multiparous client who is human immunodeficiency virus (HIV)-positive
    and asking to breast-feed her neonate as soon as possible, which of the following instructions about
    breast milk should the nurse include in the teaching plan?
  2. It may help prevent the spread of the HIV virus.
  3. It contains antibodies that can protect the neonate from HIV.
  4. It can be beneficial for the bonding process.
  5. It has been found to contain the retrovirus HIV.
A
    1. Breast milk has been found to contain the retrovirus HIV. In general, mothers are
      discouraged from breast-feeding if they are HIV positive because of the risk of possible transmission
      of the virus if the neonate is HIV negative. Breast milk does contain some immunoglobulins, but it
      does not protect the neonate from HIV infection.
      CN: Health promotion and maintenance; CL: Create
93
Q
  1. While caring for the neonate of a human immunodeficiency virus-positive mother, the nurse
    prepares to administer a prescribed vitamin K intramuscular injection at 1 hour after birth. Which of
    the following actions should the nurse do first?
  2. Bathe the neonate.
  3. Place the neonate under a radiant warmer.
  4. Wash the injection site with povidone-iodine (Betadine) solution.
  5. Wait until the first dose of antiretroviral medication is given.
A
    1. Newborns are typically bathed 2 to 4 hours after birth when their temperatures have had
      time to stabilize, but early/immediate bathing is recommended for the infants of HIV-positive mothers
      to decrease blood exposure. Placing the neonate under the radiant warmer for the vitamin K injectionis not necessary unless the neonate’s temperature is subnormal. Washing the injection site with
      povidone-iodine before giving the injection is not necessary because of the risk for possible allergy
      to iodine preparations. The first dose of zidovudine is given when the newborn is 6 to 12 hours old,
      but vitamin K is recommended to be given within an hour of birth to be most effective. Therefore the
      vitamin K should not be delayed.
      CN: Safety and infection control; CL: Synthesize
94
Q
  1. A male neonate born at 38 weeks’ gestation by cesarean section after prolonged rupture of the
    membranes and a maternal oral temperature of 102°F (38.8°C) is being observed for signs and
    symptoms of infection. Which of the following would alert the nurse to notify the primary health care
    provider?
  2. Leukocytosis.
  3. Apical heart rate of 132 bpm.3. Behavioral changes.
  4. Warm, moist skin.
A
    1. Symptoms of infection in a neonate include subtle behavioral changes, such as lethargy and
      irritability, and color changes such as pallor or cyanosis. Other symptoms include temperature
      instability, poor feeding, gastrointestinal disorders, hyperbilirubinemia, and apnea. Leukocytosis, an
      elevated white blood cell count possibly as high as 30,000 cells/mm 3 (30 × 10 9 /L) or more, may be
      normal during the first 24 hours. An apical heart rate of 132 bpm is normal. Warm, moist skin is not a
      typical sign of infection in neonates. Typically, temperature instability is common. The neonate’s
      temperature is low and the skin is cool and dry.
      CN: Health promotion and maintenance; CL: Analyze
95
Q
  1. The nurse is caring for a neonate shortly after birth when the neonate is diagnosed with sepsis
    and is to be treated with intravenous antibiotics. Which of the following will the nurse need to
    instruct the parents to do because of the neonate’s infection?
  2. Use caution near the isolation incubator and equipment.
  3. Visit but do not touch the neonate.
  4. Wash their hands thoroughly before touching the neonate.
  5. Wear a mask when holding the neonate.
A
    1. The parents of a neonate with an infection should be allowed to participate in daily care as
      long as they use good handwashing technique. This includes touching and holding the neonate. The
      parents must be careful around medical equipment to ensure proper function and around the
      intravenous site so that it is not dislodged, but being careful with the equipment is not sufficient to
      prevent transmission of microbes to the neonate. Restricting parental visits has not been shown to
      have any effect on the infection rate and may have detrimental effects on the neonate’s psychological
      development. Normally, the neonate does not need to be isolated. It is not necessary for the parents to
      wear a mask while holding the neonate. The neonate is not contagious and is receiving treatment for
      the infection.
      CN: Safety and infection control; CL: Apply
96
Q
  1. A female neonate born vaginally at term with a cleft lip and cleft palate is admitted to the
    regular nursery. Which of the following actions should the nurse do the first time that the parents visit
    the neonate in the nursery?
  2. Explain the surgical interventions that will be performed.
  3. Stress that this defect is not life-threatening.
  4. Emphasize the neonate’s normal characteristics.
  5. Reassure the parents about the success rate of the surgery.
A
    1. On the initial visit, the parents may be shocked, fearful, and anxious. Nursing care should
      include spending time with the parents to allow them to express their emotions. The nurse should
      initially emphasize the neonate’s normal characteristics. After the parents have had sufficient time to
      adjust to the neonate’s special needs, surgical interventions can be discussed. Telling the parents that
      this is not a life-threatening defect or that everything will be all right after the surgery is not helpful.
      Doing so discounts their feelings. Reassuring the parents about the success rate of the surgery can be
      done once the parents have had time to adjust to the neonate and express their emotions.
      CN: Psychosocial integrity; CL: Synthesize
97
Q
  1. After teaching the parents of a neonate born with a cleft lip and cleft palate about appropriate
    feeding techniques, the nurse determines that the mother needs further instruction when the mother
    says which of the following?
  2. “I should clean her mouth with soapy water after feeding.”
  3. “I should feed her in an upright position.”
  4. “I need to remember to burp her often.”
  5. “I may need to use a special nipple for feeding.”
A
    1. After feeding, the mouth should be cleaned with sterile water, not soapy water, to reduce
      the risk for aspiration. The cleft lip should be cleaned with sterile water to prevent crusting before
      surgical repair. The neonate needs to be fed in an upright position to prevent aspiration. The neonate
      with a cleft lip and palate commonly swallows large amounts of air during feeding. Therefore, the
      neonate needs to be burped frequently to help eliminate the air and decrease the risk for regurgitation.
      The neonate with a cleft lip and palate should be fed with a special soft nipple that fills the cleft and
      facilitates sucking.
      CN: Reduction of risk potential; CL: Evaluate
98
Q
  1. Which of the following nursing actions would support the best feeding practice for an infant
    with a cleft palate?
  2. Use an appropriate nipple and bottle set.
  3. Encourage the mother to breast-feed.
  4. Assess daily weights and wet diapers to monitor intake.
  5. Allow unlimited length of time for each feeding.
A
    1. Breast-feeding is the method of choice for all neonates. Infant feeding is more difficult for
      infants with cleft palates because of the inability to create a seal around the nipple. Maternal nipplesmost readily conform to the palate and create an adequate seal. Secondarily utilizing a nipple
      especially made for the particular cleft facilitates the seal and encourages an easier suck and
      swallow. Assessing the infant’s weight and wet diapers on a daily basis monitors the hydration and
      nutritional status. An unlimited length of time for each feeding can cause fatigue and poor weight gain.
      CN: Health promotion and maintenance; CL: Apply
99
Q
  1. A male neonate born at 36 weeks’ gestation is admitted to the neonatal intensive care nursery
    with a diagnosis of probable fetal alcohol syndrome (FAS). The mother visits the nursery soon after
    the neonate is admitted. Which of the following instructions should the nurse expect to include when
    developing the teaching plan for the mother about FAS?
  2. Withdrawal symptoms usually do not occur until 7 days postpartum.
  3. Large-for-gestational-age size is common with this condition.
  4. Facial deformities associated with FAS can be corrected by plastic surgery.
  5. Symptoms of withdrawal include tremors, sleeplessness, and seizures.
A
    1. The long-term prognosis for neonates with FAS is poor. Symptoms of withdrawal include
      tremors, sleeplessness, seizures, abdominal distention, hyperactivity, and inconsolable crying.
      Symptoms of withdrawal commonly occur within 6 to 12 hours or, at the latest, within the first 3 days
      of life. The neonate with FAS is usually growth deficient at birth. Most neonates with FAS are mildly
      to severely mentally retarded. The facial deformities, such as short palpebral fissures, epicanthal
      folds, broad nasal bridge, flattened midface, and short, upturned nose, are not easily corrected with
      plastic surgery.
      CN: Reduction of risk potential; CL: Create
100
Q
  1. Which of the following characteristics should the nurse teach the mother about her neonate
    diagnosed with fetal alcohol syndrome (FAS)?
  2. Neonates are commonly listless and lethargic.
  3. The IQ scores are usually average.
  4. Hyperactivity and speech disorders are common.4. The mortality rate is 70% unless treated.
A
    1. Central nervous system disorders are common in neonates with FAS. Speech and language
      disorders and hyperactivity are common manifestations of central nervous system dysfunction. Mild
      to severe mental retardation and feeding problems also are common. Delayed growth and
      development is expected. These neonates feed poorly and commonly have persistent vomiting until
      age 6 to 7 months. These neonates do not have a 70% mortality rate, and there is no treatment for
      FAS.
      CN: Reduction of risk potential; CL: Apply
101
Q
  1. A newborn is diagnosed with fetal alcohol syndrome. The nurse is teaching this mother what
    to expect when she goes home with her baby. The nurse determines the mother needs further
    instruction when she says which of the following?
  2. “The way my baby’s face looks now will stay that way.”
  3. “My baby may be irritable as a newborn.”
  4. “I may need some help coping with my newborn.”
  5. “My baby will be fine soon after we are home.”
A
    1. Changes seen in the facial features of newborns with fetal alcohol syndrome remain that
      way. These include epicanthal folds, whorls, irregular hair, cleft lip or palate, small teeth, and lack of
      philtrum. Newborns with fetal alcohol syndrome are usually difficult to calm and frequently cry for
      long periods of time. Parents do need assistance with caring for themselves and their infants,
      particularly with continued alcohol use. A supportive family or support systems are essential. The
      problems seen with this newborn do not go away and remain with the infant throughout life and are
      compounded when the child begins to develop mentally.
      CN: Health promotion and maintenance; CL: Evaluate
102
Q
  1. The father of a neonate diagnosed with gastroschisis tells the nurse that his wife had planned
    on breast-feeding the neonate. Which of the following should the nurse include in the preoperative
    teaching plan about feeding the neonate?
  2. The neonate will remain on nothing-by-mouth (NPO) status until after surgery.
  3. An iron-fortified formula will be given before surgery.
  4. The neonate will need total parenteral nutrition for nourishment.
  5. The mother may breast-feed the neonate before surgery.
A
    1. The parents need to know that the neonate will be kept on NPO status and will receive
      intravenous therapy before surgery. After surgery, feeding will depend on the neonate’s condition.
      Total parenteral nutrition may be prescribed after surgery, but not before. Breast-feeding may be
      started after surgery if the neonate’s condition is stable. The mother can pump the breasts until that
      time.
      CN: Reduction of risk potential; CL: Apply
103
Q
  1. The nurse is developing a plan of care for a neonate who is to undergo gastroschisis
    surgery. What should be included? Select all that apply.
  2. Prevention of hypothermia.
  3. Maintenance of fluid and electrolyte balance.
  4. Provision of time for parental bonding.
  5. Prevention of infection.
  6. Providing developmental care.
A
  1. 1,2,4. The major goals for the neonate include preventing hypothermia, maintaining fluid and
    electrolyte balance, and preventing infection. The neonate needs immediate surgery, so bonding is not
    a priority at this time. Developmental care is important and should be addressed after the closure of
    the abdominal wall defect.
    CN: Reduction of risk potential; CL: Create
104
Q
  1. While caring for a male neonate diagnosed with gastroschisis, the nurse observes that the
    parents seem hesitant to touch the neonate because of his appearance. The nurse determines that the
    parents are most likely experiencing which of the following stages of grief?
  2. Denial.
  3. Shock.
  4. Bargaining.
  5. Anger.
A
    1. The physical appearance of the anomaly and the life-threatening nature of the disorder may
      result in shock to the parents. The parents may hesitate to form a bond with the neonate because of theguarded prognosis. Denial would be evidenced if the parents acted as if nothing were wrong.
      Bargaining would be evidenced by parental statements involving “if-then” phrasing, such as, “If the
      surgery is successful, I will go to church every Sunday.” Anger would be evidenced if the parents
      attempted to blame someone, such as health care personnel, for the neonate’s condition.
      CN: Psychosocial integrity; CL: Analyze
105
Q
  1. Which of the following instructions should the nurse give to the parents of a neonate
    diagnosed with hyperbilirubinemia who is receiving phototherapy?
  2. Keep the neonate’s eyes completely covered.
  3. Use a regular diaper on the neonate.
  4. Offer feedings every 4 hours.
  5. Check the oral temperature every 8 hours.
A
    1. To prevent eye damage from phototherapy, the eyes must remain covered at all times while
      under the lights. The eye patches can be removed when the neonate is held out of the lights by the
      parents for feeding. Instead of a regular diaper, a “string” diaper or disposable face mask may be
      used to help contain loose stools, while allowing maximum skin exposure. Feeding formula or breast
      milk every 2 to 3 hours is recommended to prevent hypoglycemia and to encourage gastrointestinal
      motility. Because the phototherapy lights can overheat the neonate, the temperature should be checked
      by the axillary route every 2 to 4 hours.
      CN: Reduction of risk potential; CL: Apply
106
Q
  1. While caring for a term neonate who has been receiving phototherapy for 8 hours, the nurse
    should notify the health care provider if which of the following is noted?
  2. Bronze-colored skin.
  3. Maculopapular chest rash.
  4. Urine specific gravity of 1.018.
  5. Absent Moro reflex.
A
    1. An absent Moro reflex, lethargy, opisthotonos, and seizures are symptoms of bilirubin
      encephalopathy, which, although rare, can be life-threatening. Bronze discoloration of the skin and
      maculopapular chest rash are normal and are caused by the phototherapy. They will disappear once
      the phototherapy is discontinued. A urine specific gravity of 1.001 to 1.020 is normal in term
      neonates.
      CN: Reduction of risk potential; CL: Analyze
107
Q
  1. The nurse is caring for a neonate at 38 weeks’ gestation when the nurse observes markedperistaltic waves on the neonate’s abdomen. After this observation, the neonate exhibits projectile
    vomiting. The nurse notifies the primary health care provider because these signs are indicative of
    which of the following?
  2. Esophageal atresia.
  3. Pyloric stenosis.
  4. Diaphragmatic hernia.
  5. Hiatal hernia.
A
    1. Marked visible peristaltic waves in the abdomen and projectile vomiting are signs of
      pyloric stenosis. If the condition progresses without surgical intervention, the neonate will become
      dehydrated and develop metabolic alkalosis. Signs of esophageal atresia include coughing and
      regurgitation with feedings. Diaphragmatic hernia, a life-threatening event in which the abdominal
      contents herniate into the thoracic cavity, may be evidenced by breath sounds being heard over the
      abdomen and significant respiratory distress with cyanosis. Signs of hiatal hernia include vomiting,
      failure to thrive, and short periods of apnea.
      CN: Reduction of risk potential; CL: Analyze
108
Q
  1. The nurse is caring for a term neonate who is diagnosed with patent ductus arteriosus. While
    performing a physical assessment of the neonate, the nurse anticipates that the neonate will exhibit
    which of the following?
  2. Decreased cardiac output with faint peripheral pulses.
  3. Profound cyanosis over most of the body.
  4. Loud cardiac murmurs through systole and diastole.
  5. Harsh systolic murmurs with a palpable thrill.
A
    1. With a patent ductus arteriosus, a cardiac defect marked by a failure of the patent ductus
      arteriosus to close completely at birth, blood from the aorta flows into the pulmonary arteries to be
      reoxygenated in the lungs and returned to the left atrium and ventricle. The effect of this altered
      circulation includes increased workload on the left side of the heart and increased pulmonary
      vascular congestion. Term infants are commonly asymptomatic, but a loud, machinery-like murmur
      may be heard throughout systole and diastole. This murmur may be accompanied by a suprasternal
      thrill, and the heart may be enlarged. Decreased cardiac output with faint peripheral pulses, poor
      peripheral perfusion, feeding difficulties, and severe congestive heart failure are symptoms
      associated with severe aortic stenosis. With this defect, the aortic valve is thickened and rigid,
      leading to decreased cardiac output and reduced myocardial blood flow. Profound cyanosis over
      most of the body, fatigue on exertion, feeding difficulties, and chronic hypoxemia are associated with
      tetralogy of Fallot. With this defect, malalignment of the ventricular system results in nonrestricted
      ventral septal defects, pulmonic stenosis, overriding of the aorta, and hypertrophy of the left ventricle.
      The heart appears boot shaped. A harsh systolic murmur with a palpable thrill is associated with
      truncus arteriosus. It is marked by incomplete division of the great vessel. This is caused by a ventralseptal defect. Bounding pulses and a widening pulse pressure may also be present.
      CN: Reduction of risk potential; CL: Analyze
109
Q
  1. Assessment of a term neonate at 8 hours after birth reveals tachypnea, dyspnea, sternal
    retractions, diminished femoral pulses, poor lower body perfusion, and cyanosis of the lower body
    and extremities, with a pink upper body. The nurse notifies the primary health care provider based on
    the interpretation that these symptoms are associated with which of the following?
  2. Coarctation of the aorta.
  3. Atrioventricular septal defect.
  4. Pulmonary atresia.
  5. Transposition of the great arteries
A
    1. Coarctation of the aorta accounts for 5% to 7% of congenital heart disease. There is
      localized constriction of the aorta, at or near the insertion site of the ductus arteriosus, that increases
      afterload and decreases cardiac output. The infant with coarctation of the aorta presents with
      symptoms of poor lower body perfusion, metabolic acidosis, and congestive heart failure. Cyanosis
      is present in the lower part of the body because of decreased cardiac output. The child with a partial
      atrioventricular septal defect may be asymptomatic at birth. The symptoms in a child with a complete
      defect depend on the pulmonary artery pressure. If the pressure is high, the child will experience
      cyanosis on exertion; if it is low, congestive heart failure, manifested by tachypnea, dyspnea, and
      sternal retractions, may be present. The child with pulmonary atresia has profound (complete)
      cyanosis. On auscultation, the second heart sound is heard as a single sound. This is caused by failure
      of the pulmonary valve to develop and is accompanied by hypoplastic development of the pulmonary
      artery and right ventricle. Transposition of the great arteries is associated with complete cyanosis
      during the first few hours of life. These infants demonstrate hypoxemia with a minimal response to
      oxygen. In this defect, the pulmonary and systemic systems exist in parallel. Systemic blood
      (unoxygenated) travels to the right atrium and right ventricle, then into the aorta. Pulmonary venous
      blood (oxygenated) recirculates through the left side of the heart and through the lungs.
      CN: Reduction of risk potential; CL: Analyze
110
Q
  1. The nurse is caring for a 2-day-old neonate in the recovery room 30 minutes after surgical
    correction for the cardiac defect, transposition of the great vessels. Which of the following would
    alert the nurse to notify the primary health care provider?
  2. Oxygen saturation of 90%.
  3. Pale pink extremities.
  4. Warm, dry skin.
  5. Femoral pulse of 90 bpm.
A
    1. The normal pulse rate in a neonate is 120 to 160 bpm. Therefore, a femoral pulse rate of
      90 bpm is too low. Diminished peripheral pulses, coolness and mottling of the extremities, delayed
      capillary refill, hypotension, and decreased urine output are indicative of low cardiac output and
      poor perfusion. The neonate may be experiencing a complication of the surgery, such as blood loss or
      leaking of fluid into the interstitial space. The surgeon should be notified immediately to correct the
      diminished pulse, through either medications or transfusions. An oxygen saturation between 85% and
      100% is considered normal. The surgeon does not need to be notified unless the oxygen saturation
      falls below 85%. Pale pink extremities are considered a normal finding. If mottling or cyanosis
      develops, the surgeon should be notified immediately. Warm, dry skin is also a normal finding. If the
      skin becomes cool or appears cyanotic, the surgeon should be notified.
      CN: Reduction of risk potential; CL: Synthesize
111
Q

Assessing a neonate at 8 hours of age the nurse records the following findings on the chart below:
Vital signs.
Time 11:00.
Respiration 92, no nasal flaring, retractions grunting.
Heart rate 128, no murmur noted.
Temperature 98.9°F (37.2°C).
At 11:30, the nurse notices the neonate has central cyanosis and the respiratory rate is now 102, no
nasal flaring, no retractions, or grunting was noted and breath sounds were clear. The nurse should:
1. Change the neonate’s position.
2. Encourage the baby to cry.
3. Notify the physician.
4. Suction nose and mouth.

A
    1. The neonate is experiencing quiet tachypnea with central cyanosis which is a sign of
      possible congenital heart disease, so notifying the physician is the correct answer. The baby is
      showing no signs of increased work of breathing, except increased respiratory rate. Breath sounds are
      clear; therefore, suctioning is not necessary and may cause further distress due to trauma to the nasal
      passage. Changing the neonate’s position would have no impact on the cyanosis. Encouraging the baby
      to cry would increase the distress by decreasing oxygen consumption.
      CN: Physiological adaptation; CL: Analyze
112
Q
  1. The nurse is performing an admission assessment on a neonate and finds the femoral pulses
    to be weaker than the brachial and radial pulses. The next nursing action should be to:
  2. Call for a cardiac consult.
  3. Note and tell the physician when rounds are made.
  4. Place the neonate in reverse Trendelenburg position.
  5. Take the neonate’s blood pressure in all four extremities.
A
    1. The next nursing action in this situation would be to assess the B/P in all four extremities
      and compare the findings. A difference of 15 mm Hg in the systolic blood pressure between the arms
      and legs is an indication of a narrowed aorta. This could be an emergency and the physician needs to
      be notified as soon as the B/P data has been collected. Generally, prescribing a physician consult is
      not a nursing function. Placing the neonate in reverse Trendelenburg will only decrease the perfusionto the lower extremities.
      CN: Physiological adaptation; CL: Analyze
113
Q
113. A neonate is 4 hours of age. The nursing assessment reveals a heart murmur. The nurse
should:
1. Call the physician immediately.
2. Continue routine care.
3. Feed neonate.
4. Further assess for signs of distress.
A
    1. Further assessment for signs of distress is necessary. At 4 hours of age a transient murmur
      may be heard as the fetal shunts are closing. This is a normal finding. If no other distress is noted, the
      physician doesn’t need to be called. Result can be noted on the chart. Further assessment is needed to
      know if continuing routine care and feeding are appropriate and safe for the neonate.
      CN: Physiological adaptation; CL: Analyze
114
Q
  1. During change of shift report it was reported that the neonate was experiencing subcostal
    retractions. Identify where you would expect to see the retractions.

(with image)

A

Subcostal retractions are noted under the rib cage. Intercostal retractions are noted between the
ribs. Suprasternal retractions are found above the sternum and the substernal retractions are found
below the sternum.
CN: Physiological adaptation; CL: Apply

115
Q

Managing Care Quality and Safety
115. The nurse is making clinical rounds on a group of clients in a newborn nursery. Which infant
is at greatest risk of developing respiratory distress syndrome (RDS)?
1. A neonate born at 36 weeks’ gestation.
2. A neonate born by cesarean section.
3. A neonate experiencing apneic episodes.
4. A neonate who is 42 weeks’ gestation

A

Managing Care Quality and Safety
115. 1. The preterm infant is at greatest risk for developing RDS as the lungs are immature and
unable to produce adequate surfactant to reduce the surface tension in the lungs and promote the
stability of the alveoli. The neonate born at 42 weeks’ gestation is at greater risk of having meconium-
stained amniotic fluid and developing meconium aspiration syndrome. The neonate born by cesarean
section is at greater risk of fluid retention in the lungs as the lungs were not compressed during the
birth process as they are in a vaginal birth. Generally, these infants are able to overcome this situation
with adequate crying. Apneic episodes in infants may be related to several conditions, including
hypothermia, sepsis, and hypoglycemia, or a result of a rapid increase in the infant’s temperature.
CN: Management of care; CL: Analyze

116
Q
  1. The nurse has received a shift report on a group of newborns. The nurse should make rounds
    on which of the following clients first?
  2. A newborn who is large for gestational age (LGA) who needs a repeat blood glucose prior to
    the next feeding in 15 minutes.
  3. A neonate born at 36 weeks’ gestation weighing 5 lb (2,270 g) who is due to breast-feed for the
    first time in 15 minutes.
  4. A neonate who was born 24 hours ago by cesarean section and had a respiratory rate of 62, 30
    minutes ago.
  5. A newborn who had a borderline low temperature and was double-wrapped with a hat 1⁄2 hour
    ago to bring up the temperature.
A
    1. The nurse should make rounds and first assess the neonate with the respiratory rate of 62.
      The respiratory rate is out of the normal range and needs reevaluation. The newborn who is LGA still
      has 15 minutes before being due for the feeding and much can be accomplished by the nurse in that
      time. A 36-week newborn weighing 5 lb (2,270 g) will need to be fed on time to maintain the blood
      glucose level. The nurse should next assess the infant with a borderline low temperature to determineif his body temperature is increasing.
      CN: Management of care; CL: Synthesize
117
Q
  1. The nurse managing the admission nursery is beginning the shift. There are 2 infants under
    the care of a primary staff nurse and are remaining in the nursery while their mothers sleep. One
    newborn is waiting to be transferred to the special care nursery (SCN) with a diagnosis of possible
    sepsis. The SCN cannot accept a transfer for 30 minutes. The nurse has been notified that another
    infant has been born and is breathing at a rate of 80 bpm and needs to be admitted to the nursery.
    There are also two infants who are waiting for social services to determine follow-up. There can be
    no other additions to the nursery until at least one newborn leaves the area. How should the nurse
    manage this situation?
  2. Ask the nurses in SCN if they can take the newborn with possible sepsis now.
  3. Ask the primary staff nurses to take their babies back to the sleeping mothers’ rooms.
  4. Call social services to determine if either of the babies who are waiting to be discharged is
    ready to leave.
  5. Ask the nurse with the infant who is breathing at 80 bpm to wait 1⁄2 hour.
A
    1. The nurse should manage this situation by asking the staff nurses to take at least one baby
      back to a room with a sleeping mother. This would allow the babies who are in need of care in the
      nursery to remain there. To maintain safety, the SCN cannot admit a client until they are prepared and
      have the staffing for this infant to be transferred. Social services can be called to determine if either
      of the newborns who have been referred to them can be discharged, but releasing these infants from
      the nursery will take several hours. It is unsafe to keep the infant with a respiratory rate of 80s
      waiting for a bed.
      CN: Management of care; CL: Create
118
Q
  1. The nurse in a postpartum couplet room is making rounds prior to ending the shift. Which of
    the following indicate that the safety needs of the clients have been met? Select all that apply.
  2. Infant lying on abdomen.
  3. Security tags in place.
  4. Identification system on mother and infant.
  5. Bulb syringe within sight.
  6. Someone in room able to care for infant.
  7. Infant in the mother’s arms, both asleep.
A
  1. 2,3,4,5. A hospital-specific security system is the standard of care to prevent neonatal
    abduction. The bulb syringe should be visible and easily accessible to both the mother and the nurse
    in case of choking. Someone should remain in the room who is able to safely care for the infant. This
    may be the mother or a family member if the mother is physically not able to care for the infant. The
    infant should be lying on the back or side, rather than the abdomen to prevent Sudden Infant Death
    Syndrome (“back to sleep”). Infant falls from a mother’s bed are a serious safety problem. The infant
    should be in the mother’s arms or in the crib rather than lying on a bed, even with the side rails up, as
    an infant can slip through the rails. A sleeping mother is not aware of the status of the infant who can
    easily fall out of her arms; the infant can be in the mother’s arms only if she is awake.
    CN: Safety and infection control; CL: Evaluate
119
Q
  1. After receiving change of shift report in the normal newborn nursery, which neonate should
    the nurse see first?
  2. Neonate A, 1/2 hour of age with intermittent respiratory grunting.2. Neonate B, 4 hours of age with a blood glucose of 30 (1.7 mmol/L).
  3. Neonate C, 12 hours of age with a temperature of 97.4 (36.4 °C).
  4. Neonate D, 24 hours of age with no urine output for past 12 hours.
A
    1. The blood glucose of 30 (1.7 mmol/L) is the most critical. Glucose is the only fuel that the
      brain can use. It is important to protect the central nervous system and levels less than 40 (2.2
      mmol/L) in a neonate indicate hypoglycemia. Grunting at 1⁄2 hour of age may be normal transitioning to
      extrauterine life. A temperature of 97.4 (36.4 °C) is only slightly low for a neonate at this age. Ninety
      five percent of all neonates will void at least once in the first 24 hours. This is not unusual at this age.
      CN: Management of care; CL: Analyze
120
Q
  1. The newborn nurse has just received shift report about a group of newborns and is to
    receive another admission in 30 minutes. In order to provide the safest care and plan for the new
    admission, the nurse should do which of the following in order of first to last?
  2. Move quickly from room to room and assess all clients.
  3. Check the room to which the new client will be admitted to be sure all supplies and equipment
    are available.
  4. Log on to the clinical information system and determine if there are new prescriptions.
  5. Review notes from shift report and prioritize all clients; make rounds on the most critical first.
A

120.
4. Review notes from shift report and prioritize all clients; make rounds on the most critical first.
1. Move quickly from room to room and assess all clients.
3. Log on to the clinical information system and determine if there are new prescriptions.
2. Check the room to which the new client will be admitted to be sure all supplies and equipment
are available.
Based on the report given by the preceding nurse, the nurse should plan to prioritize all clients
and first make rounds on the client needing the highest level of nursing care. The nurse can then make
rounds on all other clients. The nurse can then check for new prescriptions and, finally, inspect theroom in which the next client will be admitted to be sure all of the equipment is available.
CN: Management of care; CL: Synthesize

121
Q
  1. The charge nurse in the newborn nursery has an unlicensed assistive personnel (UAP) with
    her for the shift. Under their care are 8 babies rooming in with their Moms, and 1 infant in the nursery
    for the night on tube feedings. There is a new client whose infant will be brought to the nursery in 15
    minutes. Which tasks would the RN assign to the UAP? Select all that apply.
  2. Newborn admission.
  3. Vital signs on all stable infants.
  4. Tube feeding.
  5. Document feedings of infants.
  6. Record voids/stools.
  7. Bath and initial feeding for new admission.
A
  1. 2,4,5. The role of the UAP allows this member of the health care team to take vital signs on
    patients, record feedings, and voids and stools of infants according to hospital guidelines. The
    newborn assessment is completed by a licensed care provider as is the tube feeding. Bathing of the
    newborn is within the scope of practice for the nursing assistant but the initial assessment of patency
    of the gastrointestinal tract, which is initiated by the first feeding, is within the scope of licensed care
    providers. If there is a trachea esophageal fistula, this is the time when it may become evident.
    CN: Management of care; CL: Apply